gmat questions – Online GMAT Prep Blog by PrepScholar /gmat/blog GMAT Prep Online Guides and Tips Thu, 22 Aug 2024 16:21:34 +0000 en-US hourly 1 https://wordpress.org/?v=4.8.3 How the 7 Best GMAT Apps Can Improve Your Score /gmat/blog/best-gmat-apps/ Tue, 02 Oct 2018 16:00:36 +0000 http://gmat.psblogs.com/?p=1465 GMAT apps are an undeniably appealing way to study for the GMAT: they’re convenient, fun, and a great excuse to play with your phone. Good ones can be a great addition to your study plan, but they aren’t enough on their own. This article is a complete guide to GMAT apps that will help you … Continue reading "How the 7 Best GMAT Apps Can Improve Your Score"

The post How the 7 Best GMAT Apps Can Improve Your Score appeared first on Online GMAT Prep Blog by PrepScholar.

]]>
beautiful-15742_1280

GMAT apps are an undeniably appealing way to study for the GMAT: they’re convenient, fun, and a great excuse to play with your phone. Good ones can be a great addition to your study plan, but they aren’t enough on their own.

This article is a complete guide to GMAT apps that will help you lean how to use GMAT apps effectively, rather than letting them become a time-wasting distraction. I’ll go over the qualities that make a GMAT app worth using, the seven best GMAT apps available, and the best ways to incorporate apps into your GMAT prep.

 

GMAT Apps: An Overview

You can use apps for almost anything, and GMAT prep is no exception. GMAT prep apps may include practice tests, additional practice questions, drills targeting particular skills, lessons, and other content to help you prepare for the exam and track your progress.

GMAT apps that you download to your tablet or phone can help you get ready for the test in an interactive and even fun way. They aren’t enough as a comprehensive source of GMAT prep, but they can be a great addition to your other practice materials. If you’re looking for an affordable, comprehensive online prep program, check out PrepScholar GMAT. The course includes access to over 1,000 practice questions and 30 lessons covering all the essential GMAT skills.

 

GMAT apps can be a great, free or inexpensive addition to your GMAT prep.
GMAT apps can be a great, free or inexpensive addition to your GMAT prep.

 

Why GMAT Apps Are Not Enough

GMAT apps can be a great resource, but they’re not enough for adequate GMAT prep or to achieve your target score. You’ll need to use more comprehensive GMAT study resources like practice tests and books if you want to improve.

Firstly, most apps aren’t catered to your particular weaknesses and strengths. A few apps track your progress, but not in in-depth ways.

Secondly, most of the GMAT apps don’t have enough realistic practice questions to be effective as your primary or only GMAT prep resource. Many of them only have a few, or the practice questions aren’t organized by question type, so it’s difficult to target specific skills or concepts.

Finally, GMAT apps don’t tend to include detailed answer explanations for practice questions, meaning you may not be able to pinpoint why you missed the questions and determine how to improve for the future.

 

What Makes a Good GMAT App?

Let’s take a look at the qualities that make a GMAT app effective for use as part of your overall prep.

 

#1: Realistic Practice Questions

If a GMAT app features practice questions, they should be similar in content, tone, difficulty, and length to those on the actual GMAT. The more realistic, the better the questions are for your GMAT prep.

 

#2: In-Depth Lessons and Content

If a GMAT prep app contains skill-building lessons, they should be detailed, breaking down each question type or necessary skill in depth. You should be able to come away from a lesson with an understanding of a topic you previously didn’t know much about.

 

#3: User-Friendly

Any GMAT prep app you choose to download should be easy to understand, use, and navigate. It should be clear how to find information you need, and easy to select answers on practice questions. Some apps are difficult to read due to issues with the text or visuals.

 

The best GMAT apps are easy to navigate and user friendly.
The best GMAT apps are easy to navigate and user-friendly.

 

The Top 7 GMAT Apps

Let’s go over some of the best GMAT apps available for download. For each GMAT prep app, I’ll go over its cost, its content, and its advantages and disadvantages.

 

Manhattan Prep GMAT

The Manhattan Prep GMAT app has lots of quality, varied content, including 1,110 quantitative and verbal practice questions; 1,556 quantitative, vocab, idioms, and sentence correction drills and quizzes; strategies for each question type; general GMAT study tips; math and verbal glossaries; MBA tips; and a section that tracks your progress, including your average timing on each practice question.

You can try out the Manhattan Prep GMAT app for free for 24 hours. After that, if you want to upgrade to premium, it will cost you $29.99. Otherwise, you can stay at the ‘basic’ level, which contains fewer customization options and fewer practice questions.

The Manhattan Prep GMAT app is the most comprehensive one out there, with high quality practice questions and lots of drills in both basic and advanced quant (hard to find). The idiom and vocabulary drills are also helpful for non-native English speakers. Questions are divided by difficulty as well as skill type, so the app will help you hone in on your weaknesses effectively.

 

Ready4 GMAT (Prep 4 GMAT)

The basic version of Ready4 GMAT is free with an account, while the premium version (which includes over 1,000 practice questions, 500 flashcards, GMAT prep and vocab quizzes, and an in-depth prep course, including a full GMAT practice test) costs $19.99.

The free version has a great deal of content, though, including an assessment test with 10 practice GMAT questions and an in-app GMAT prep course, with lessons on each GMAT section, fundamental skills like grammar, quizzes, and drills on specific question types. The only downside is that you can’t access a lesson without completing the one before it.

Ready4 GMAT also has some unique features, such as a “school matcher” that lets you customize your list of prospective business schools according to your personal demographics, preferences, academic background, GMAT score, and goals. If you complete it, you can enter to win a $1,000 Ready4 scholarship to business school.

 

GMAT Math Flashcards and GMAT Idiom Flashcards (Magoosh)

Both of these helpful, targeted apps from Magoosh are free. They don’t contain full-length practice questions; instead, they allow you to test fundamental skills that will help you on the quant and verbal sections.

The GMAT Math Flashcards ask about basic math concepts or simple calculations (such as determining 20% of a given number in your head). They can help you practice estimation, which is important for the quant section since you won’t have access to a calculator. The set contains 19 algebra practice questions; 38 questions on fractions, ratios, and percents; 51 geometry questions; two sections of 33 and 32 questions, respectively on number properties; 24 statistics and probability questions; and six sets of miscellaneous practice, containing 36-37 questions each.

Each GMAT Idiom Flashcard contains two versions of a sentence with variations on an idiom; you select which one is correct. After you virtually ‘flip’ the card, you’ll see the correct answer and will be asked whether you were familiar with the correct idiom. The cards are divided into two basic and two advanced sections; they track your mastery and will test you on unfamiliar idioms as you continue to practice.

These are helpful flashcards on the go, or to target a particular math skill. The idiom list will be especially helpful for non-native English speakers. However, they don’t contain actual GMAT practice questions, so they won’t be enough for substantial GMAT prep.

 

Mobile flashcards are a helpful resource for GMAT prep.
Mobile flashcards are a helpful resource for GMAT prep.

 

GMAT Prep Course (Veritas Prep)

With a free GMAT On Demand trial account, you can access over 24 hours of instructional videos (a ‘self-guided course’) from Veritas Prep that take you through every section of the GMAT and every skill you’ll need to master. With your trial account, you can also access various online Veritas Prep practice materials, including a computer-adaptive GMAT practice test and GMAT practice questions, for seven days.

Veritas Prep lessons are high-quality and informative, with instructors guiding you through each GMAT question type and concept in detail. Each lesson (such as ‘arithmetic calculations,’ the first section of the arithmetic lesson) is around 20-30 minutes.

While you’ll need to purchase the full course to get permanent access to the Veritas Prep resources, you can use this app for an overview of the test and the skills you’ll need to learn, especially at the beginning of your prep process.

 

Economist GMAT Tutor

You’ll have to create an account to access a free trial Economist GMAT Tutor app, which includes one free tutor session, opportunities to ask questions of tutors trained by The Economist, and a simulated GMAT test, as well as lessons that delve into specific GMAT skills.

The biggest advantage of this app are the in-depth lessons. Each lesson breaks down the various aspects of a particular section or question type in more detail than the majority of app-based GMAT lessons. One disadvantage, besides the obvious drawback of most of the materials only being available for purchase, is that you’ll have to complete one lesson before moving on to the next. So if you’re looking to quickly target a specific skill, this isn’t the best app for your GMAT prep.

 

GMAT: Practice, Prep, Flashcards (Varsity Tutors)

The Varsity Tutors GMAT: Practice, Prep, Flashcards app is completely free. You can find flashcards organized by concept, diagnostic tests, and GMAT practice tests and quizzes.

The Varsity Tutors app’s primary strength is its organization. While many apps organize questions according to general categories, like ‘quant section’ or ‘advanced quant,’ Varsity Tutors’ concept categories are hyper-specific and usually include several subcategories. This means you can target your weaknesses much more effectively using Varsity Tutors than on many other apps.

The quant practice questions in the diagnostic tests are a little on the easy side, and the app itself can be a little confusing to navigate. Other than that, this is a comprehensive and effective app to switch up your method of GMAT prep.

 

GMAT Exam Prep (Pocket Prep, Inc.)

The GMAT Exam Prep app from Pocket Prep offers flashcards, practice questions, and personal statistics on your progress. You can also use the app to challenge other users in GMAT practice question duels.

The app’s benefits are its variety (flashcards on everything from analytical writing-specific vocabulary to advanced quant) and its ability to track your progress over time. The app schedules you for reviews of certain GMAT concepts and skills based on your weaknesses. The downsides? There are some distracting typos in questions and headings, and the ads can be annoying as well.

You can access this app free with an account or upgrade it for a fee. The Basic version (with no ads and offline study available) is $4.99, while the Premium version (which will store all the app’s data in the Cloud) is $9.99.

 

GMAT apps can help you develop strategies to improve your GMAT score.
GMAT apps can help you develop strategies to improve your GMAT score.

 

How to Use GMAT Apps

There are a few ways to use GMAT apps that will maximize their usefulness for your exam prep.

 

#1: To Focus On Your Weaknesses

The drills in GMAT prep apps can sometimes help you get an extra boost in particularly weak areas. Apps with short quizzes or mini-lessons centered around certain question types or foundational skills can be helpful as an extra resource for your trouble spots.

 

#2: To Add Practice Questions

Additional GMAT practice questions, especially free ones, are always a big plus during prep. As long as the questions in the apps you choose are realistic and high-quality, more practice is always better.

 

#3: On the Go

Since GMAT apps can be downloaded to a phone or tablet, they can be a great resource for extra practice while you’re out and about or if you only have a short time.

Also, it can be useful to study in a variety of ways. Prepping using a different form of technology can motivate you and allow you to engage with the material in a new way.

 

Wrap-Up: Using GMAT Apps

GMAT apps can be a great addition to your overall prep. They can help you switch it up and learn or practice using a new interface and in a new way.

While they’re not enough as a comprehensive source of practice, you should consider adding a GMAT prep app (or two!) to your study plan.

 

Engaging your brain in a new way is one of the possible pros of using apps to prepare for the GMAT.
Engaging your brain in a new way is one of the possible pros of using apps to prepare for the GMAT.

 

What’s Next?

Find additional practice questions in our extensive collection of GMAT sample questions.

In How to Prepare for the GMAT, you can find a more comprehensive guide to GMAT prep.

If you’re a non-native English speaker or looking to improve your verbal score, our guide to GMAT idioms will help you prepare for sentence correction and critical reasoning questions on the GMAT.

The post How the 7 Best GMAT Apps Can Improve Your Score appeared first on Online GMAT Prep Blog by PrepScholar.

]]>
The Best GMAT IR Practice: 200+ Questions for Your Prep /gmat/blog/gmat-integrated-reasoning-practice-questions/ Mon, 02 Apr 2018 15:00:55 +0000 http://gmat.psblogs.com/?p=1213 For many students, there’s an air of mystery surrounding the GMAT Integrated Reasoning section.  Designed to test real-world skills, the IR section can seem more complicated to study for than the other sections of the exam. In this guide, I’ll help you find the best tools for GMAT integrated reasoning practice. First, I’ll talk about what the IR … Continue reading "The Best GMAT IR Practice: 200+ Questions for Your Prep"

The post The Best GMAT IR Practice: 200+ Questions for Your Prep appeared first on Online GMAT Prep Blog by PrepScholar.

]]>
boy-1300226__480

For many students, there’s an air of mystery surrounding the GMAT Integrated Reasoning section.  Designed to test real-world skills, the IR section can seem more complicated to study for than the other sections of the exam. In this guide, I’ll help you find the best tools for GMAT integrated reasoning practice.

First, I’ll talk about what the IR section tests and the types of questions you’ll see on it. Next, I’ll talk about what you need to prepare for the IR and what you should look for in your IR practice materials. Then, I’ll review both official and unofficial practice materials so that you have a good starting place to begin your practice. Finally, I’ll give you some tips for making the most out of your Integrated Reasoning GMAT practice.

 

What’s on the GMAT Integrated Reasoning Section?

The GMAT Integrated Reasoning section is non-adaptive (meaning the difficulty level of questions doesn’t vary depending on how you do), contains 12 questions, and lasts for 30 minutes. The IR section is scored from 1 to 8, in single-digit increments. Like the Analytical Writing Assessment, your IR score is separate from your Quantitative and Verbal scores and doesn’t factor into your total GMAT score.

IR questions a more open-ended than normal multiple choice questions. Instead of simply selecting the one right answer of five options, you may choose one, two, three, four, or even all five answers as correct. IR questions ask you to use both your Verbal and Quantitative skills together. You’ll have to analyze data in a number of forms (words, charts, tables, etc) and pull out insights from each.

There are four types of GMAT IR questions:

  • Two-part analysis: these questions are often wordy and have small tables attached to them. You’ll be asked to choose two correct answers out of five or six options.
  • Multi-source reasoning: these questions present information from, you guessed it, multiple sources. You’ll navigate through and analyze multiple sources of data.
  • Graphic interpretation: these questions require you to analyze the information presented in a graph or a chart. You’ll have two questions, with answer choices presented in drop down menu format.
  • Table analysis: data in these questions is presented in a sortable table along with three questions. To answer these questions correctly, you’ll need to differentiate between essential and nonessential information.

 

What Should I Look for in GMAT Integrated Reasoning Practice?

checklist-1622517_1280

 

As a test-taker, it can be hard to figure out which GMAT integrated reasoning practice questions are right for you. Here are some questions to ask yourself when considering which materials to use.

 

#1: Do the Questions Have the Same Format as Real GMAT IR Questions?

GMAT IR questions have unique formats that are unfamiliar for most students and can be confusing. The only way to ensure you know how to approach the questions on test day is to practice with questions that look like those you’ll see on the real GMAT IR section until you’re comfortable with with their style. As such, it’s vital that your GMAT IR practice questions are the same four types and closely resemble those you’ll see on the test.

 

#2: Is the Resource Computerized?

You’ll take the GMAT on a computer, so it makes sense to practice on a computer as much as possible before test day. This is especially important for IR practice, since the questions require you manipulate data and interact with answers in ways that aren’t possible on paper.

 

#3: Does the Difficulty Level of the Practice Questions Match the Real GMAT?

While the Integrated Reasoning section isn’t adaptive, you will see questions at different difficulty levels (easy, medium, and hard). When practicing, you’ll want to make sure that your GMAT IR practice questions cover all difficulty levels so you’re prepared to solve whatever questions you see on test day.

You should also beware that unofficial questions can sometimes be much easier or harder than the questions you’ll actually see on the GMAT. If you notice that you’re doing much better or worse than usual on a specific set of practice questions, consider that they may not be very accurate.

 

#4: Are Practice Questions Grouped by Skill or Difficulty Level?

Targeted practice, when you focus your practice session on one specific skill or question type, is one of the most effective ways to prepare for the GMAT. If you decide to focus on solving only graphic interpretation questions for one hour-long session, you’ll learn more than if you break that same one-hour session into shorter, 15-minute sections for all four question types. You might also want target your practice by difficulty level, for instance working at solving easy questions in a shorter amount of time.

Resources that allow you to sort questions by difficulty level or skill are extremely valuable, since they make creating targeted practice sets much simpler.

 

Official GMAT Integrated Reasoning Practice

student-849825__480

Using official GMAT integrated reasoning practice questions is a great way to ensure that you’re preparing with high-quality practice questions. The questions in GMAT official resources are actual, retired GMAT questions, written by the same people who write the real GMAT.

 

The Official Guide for GMAT Review 2017 ($28.89)

This print guide, written by GMAC, is an official study guide that covers all sections of the GMAT. The guide comes with a chapter dedicated to the Integrated Reasoning section, which reviews the question types and discusses strategies for solving problems. The guide also comes with access to an online Integrated Reasoning companion, which includes more information about the IR section as well as 50 online IR practice problems.

Overall, it’s a great place to start with your GMAT prep, including for IR.

GMATPrep (Free)

This online software gives you access to two full-length practice computer-based tests, with the option of purchasing up to six total CATs. Each of the practice tests includes one IR section, with 12 questions.

Beyond the CATs, GMATPrep includes an additional 15 free IR practice questions.

 

IR Prep Tool ($19.99)

This online software includes 48 IR questions, answer explanations, and customizable question sets. This resource is great because it lets you create your own sets of GMAT IR practice questions, sorting by question type and difficulty. It also lets you practice questions in either study mode (untimed) or exam mode (timed).

If you really want to focus on GMAT IR practice, this is one of the best tools available, but it’s fairly expensive for just a few dozen questions, especially compared to the full official guide.

 

Unofficial GMAT Integrated Reasoning Practice

Because the Integrated Reasoning section is the newest section on the GMAT, there are relatively few resources out there, especially if you compare the few GMAT Integrated Reasoning practice questions to the numerous Quant or Verbal questions. Even so, there are some high-quality GMAT IR practice materials out there that you can use to supplement the official resources in your prep.

 

Veritas Prep Integrated Reasoning Sample Questions

Veritas Prep has developed a number of Integrated Reasoning practice questions, and offers 14 of them free-of-charge on their website. Veritas Prep is known for having high-quality practice questions and this resource gives you realistic examples of all four types of questions that you’ll find on the IR section. The questions are also accompanied by in-depth explanations that explain why each answer is correct or incorrect.

 

GMAT Club’s E-Book Bundle

GMAT Club, an online forum that provides support and advice for GMAT test takers, has compiled a list of all free e-books and practice question sets that address the IR section. This resource is great for test takers who are looking for materials that include both instruction and practice questions. Note that these downloadable resources are available as PDFs, so you won’t be able to solve questions online.

 

GMAT Pill’s Integrated Reasoning Sample Questions

GMATPill has compiled nearly 200 IR sample questions that you can practice online. These questions are broken down into question type, with specific information on skills tested and solve rate, so you can see how hard each question is.

The online format is useful, because you’ll get used to solving the questions on a computer, but the web-based interface doesn’t look anything at all like the interface on the real GMAT. Keep that in mind as you’re practicing.

 

800score.com’s Integrated Reasoning Sample Questions

800score.com offers demos of the four question types in an online format that has a very similar interface to the actual GMAT. The site also offers instructional videos and other resources you can use in your prep.

Unfortunately, there are only four GMAT integrated reasoning practice questions here, but they’re of high quality and worthwhile for their similarity to actual GMAT IR questions.

 

4 Tips for Making the Most of Your GMAT Integrated Reasoning Practice

stopwatch-153398__480

 

It’s important to make sure that you’re making the most out of your GMAT test prep by practicing smart. Here are some tips for how to make sure you’re making your GMAT IR prep valuable.

 

#1: Practice Your Pacing

The GMAT IR section is short – only 30 minutes to solve 12 complex, multi-step questions. It’s important to consider timing when you’re practicing. You’ll want to pay attention to how long it takes you to solve problems, particularly during practice tests, when you’ll be answering all 12 questions in one sitting. As you practice, place time limits on yourself and work to see how quickly you can solve problems.

 

#2: Put Away the Calculator

The IR section doesn’t let you use your own calculator, though you’ll have access to an online calculator with basic functions. Practice using the online calculator with your sample questions, or using a simple four-function calculator on your phone that mimics the simple calculator you’ll use on the test. By only using the resources you’ll actually have on the GMAT, you’ll ensure you’re comfortable and prepared on test day.

 

#3: Dedicate Time to IR Practice

Many test takers skip preparing for the IR section in favor of spending more time on the Quant and Verbal sections, since they’re considered more important. But as the IR section becomes more seasoned, business schools are considering IR scores more. Especially since the IR section is designed to simulate skills you’ll need in the real world, it’s important to make sure you do well on it!

While you’ll likely spend more time prepping for the Quant and Verbal sections, build in time to focus on IR. Make sure you take the IR section on every practice test you do and spend a few dedicated study sessions on IR questions.

 

#4: Familiarize Yourself With the Questions

The IR questions look different from every other question type on the GMAT. Spend time with the practice questions so that you familiarize yourself with the visual nature of these questions, as well as with the practice of choosing multiple right answers. Doing so will help you feel more confident on test day. It will also save you time, as you’ll be able to easily recognize question types and know what to when you take the actual exam.

 

What’s Next?

Learn more about the other sections of that GMAT by reading our guides on GMAT Quant and GMAT Verbal.

Is the GMAT Total score the only one that matters? Read our guide to find out how the total score is calculated and how business schools weight the different scores.

Take a look at our massive collection of GMAT sample questions to start your prep with a huge selection of practice questions for all four sections,

The post The Best GMAT IR Practice: 200+ Questions for Your Prep appeared first on Online GMAT Prep Blog by PrepScholar.

]]>
GMAT Quantitative: 10 Tips to Master the Math Section /gmat/blog/gmat-quantitative-review-questions-tips/ Mon, 12 Feb 2018 17:00:22 +0000 http://gmat.psblogs.com/?p=1266 The GMAT quantitative section is probably the most notorious and daunting section of the exam. There are many myths surrounding the quant section of the GMAT, such as that it tests extremely advanced math concepts or that it’s impossible to achieve a perfect score. However, by building your understanding of the quant section through careful preparation, … Continue reading "GMAT Quantitative: 10 Tips to Master the Math Section"

The post GMAT Quantitative: 10 Tips to Master the Math Section appeared first on Online GMAT Prep Blog by PrepScholar.

]]>
night-theme-numbers-black-and-white-pen-74228

The GMAT quantitative section is probably the most notorious and daunting section of the exam. There are many myths surrounding the quant section of the GMAT, such as that it tests extremely advanced math concepts or that it’s impossible to achieve a perfect score. However, by building your understanding of the quant section through careful preparation, it’s more than possible to do well on this challenging part of the GMAT.

In this complete GMAT quantitative review, I’ll be giving you an in-depth look into the format of the section and what skills are tested on it. I’ll also take you through a detailed look at the two types of GMAT quant questions (data sufficiency and problem solving). Finally, I’ll give you tips that’ll help you achieve success in your test preparation and on test day.

 

GMAT Quantitative Overview

The GMAT quant section tests your ability to analyze data and draw conclusions using reasoning skills.  There are 31 multiple choice questions on this section that test your abilities in these areas, and you’ll have 62 minutes to complete the section. The quant section is the third section of the test, after the analytical writing assessment and integrated reasoning sections.

Like the two previous sections on the GMAT, you’ll take the quant section on a computer. However, unlike the integrated reasoning and analytical writing assessment, the quant section is adaptive. What that means is that the difficulty of the questions you get will be adjusted as you get questions right or wrong. If you’re struggling, you’ll receive easier questions. If you’re doing well, you’ll receive more difficult questions. Your score on the quant section is determined by three things: the number of questions you answer, the number of questions you answer correctly, and the difficulty of the questions you answer.

Quant scores technically range from 0 to 60, but the range in which people actually score is 6 to 51. Your quant score also contributes to your GMAT total score, which ranges from 200 to 800.

 

What’s Tested on the GMAT Quantitative Section?

math-1547018__480

The quant section tests your content and analytical knowledge of basic math concepts, such as arithmetic, algebra, and geometry. Contrary to popular belief, the GMAT quant section doesn’t test on advanced math concepts. In fact, you only need to know high-school-level math for the GMAT quant section. You’ll see the following concepts on the test:

  • algebraic equations and inequalities
  • arithmetic
  • decimals
  • percentages
  • ratios
  • exponents and square roots
  • geometry and coordinate geometry
  • integers
  • factors
  • multiples
  • number lines
  • variable operations

Rather than testing your knowledge of complex mathematical concepts, the GMAT wants to see how you apply your knowledge of basic math concepts on two types of questions: data sufficiency and problem solving. While both question types will require you to do the same kinds of math, they’re testing very different skills. The problem solving questions test how well you can figure out the answers to different kinds of numerical problems, such as knowing how much commission to give on the sale of a good. The data sufficiency questions test your ability to determine whether information is adequate enough to solve problems. We’ll talk more about these two types of questions in the next section.

 

The 2 Types of GMAT Quant Questions

There are two types of GMAT quantitative questions: data sufficiency and problem solving. In this section, I’ll go into more detail about the format of each question type and give you an example problem and solution for each.

If you’d like to get more in-depth information about strategies for either of these sections, check out our detailed data sufficiency and problem solving guides (coming soon).

 

Data Sufficiency

Data sufficiency questions are multiple choice questions that come with two statements of data. Your job is to figure out whether or not the statements provide sufficient data to answer the question. Then, you’ll have to decide whether one or both statements, by themselves or together, give you enough information to answer the problem. You’ll have the same five answer choices for every data sufficiency question:

  1. Statement (1) ALONE is sufficient, but statement (2) alone is not sufficient.
  2. Statement (2) ALONE is sufficient, but statement (1) alone is not sufficient.
  3. Both statements TOGETHER are sufficient, but NEITHER statement ALONE is sufficient.
  4. EACH statement ALONE is sufficient.
  5. Statements (1) and (2) TOGETHER are NOT sufficient.

Let’s look at what a data sufficiency question looks like in practice.

 

Data Sufficiency Sample Question

Here’s an example of a data sufficiency sample question that’s retired from an actual GMAT.

Is the average (arithmetic mean) of x and y greater than 20?

  1. The average (arithmetic mean) of 2x and 2y is 48.
  2. x = 3y
  1. Statement (1) ALONE is sufficient, but statement (2) alone is not sufficient.
  2. Statement (2) ALONE is sufficient, but statement (1) alone is not sufficient.
  3. Both statements TOGETHER are sufficient, but NEITHER statement ALONE is sufficient.
  4. EACH statement ALONE is sufficient.
  5. Statements (1) and (2) TOGETHER are NOT sufficient.

First, let’s look at the features of this data sufficiency question. Notice how the question comes first (“Is the average (arithmetic mean) of x and y greater than 20?”). After that, the two statements are listed. Your job is to figure out whether each of those statements is enough to solve the question, either by themselves or together. Let’s look at the solution for that problem.

 

Data Sufficiency Sample Question Explanation

The first step for any question on the quant section is to understand what the question is asking you to find. This question is asking you to determine the mean of x and y. The mean (or average) can be expressed as the sum of both numbers divided by 2. So, you can express the mean of x and y in either of these equations:

(x + y)/2 > 20

x + y > 40

Now, you’ll want to turn your attention to each statement. Let’s look at statement 1 first. For this statement, you’ll need to express the statement as an equation and then simplify to solve for x + y. First, let’s figure out your equation. The statement says that the average of 2x and 2y is 48. That information yields this equation:

(2x + 2y)/2 = 48

Next, you can simplify to isolate x and y.

2x/2 + 2y/2 = 48

x + y = 48

Think back to the first inequalities. We said that we can express the mean of x and y as either (x + y)/2 > 20 or x + y > 40. In the math we just did for statement 1, we simplified the equation to show that x + y = 48. 48 is greater than 40, which satisfies the requirement that x + y > 40. That means that the information is sufficient.

Now, let’s look at our second statement. Whenever you’re solving a data sufficiency question, you want to first solve the statements by themselves before considering them together. We solved statement 1 by itself, now we’re solving statement 2 by itself.

Statement 2 says that x = 3y. That means that we can substitute for x in our original inequality.

x + y > 40

3y + y > 40

Remember, x = 3y. So, in my second equation I substituted in 3y for x. Now, I can combine like terms.

4y > 40

y > 10

So, solving this inequality shows us that y is greater than 10. But let’s think back to what the question is asking us. The question is asking us if x + y > 40. From solving this inequality, we don’t have enough information about y or about x to see if x + y > 40.

You can also solve statement 2 by substituting values for x and y that satisfy the condition x = 3y. For instance:

If y = 7 and x = 21 then (x + y)/2 = 14, which is NOT greater than 20.

If y = 40 and x = 120, then (x + y)/2 = 80, which IS greater than 20.

Because there’s the possibility to get an answer greater than OR less than 20 for statement 2, we’ll need more information to solve the problem.

The correct answer to this sample question is, then, is A: Statement (1) ALONE is sufficient, but statement (2) alone is not sufficient.

 

Problem Solving

question-423604__480

Problem solving questions are multiple-choice questions that test high school math skills. These questions are much more traditional and straight-forward than data sufficiency questions. For these questions, you’ll need to use basic math concepts from topics like arithmetic, algebra, and geometry to solve math problems. Let’s take a look at a problem solving sample equation.

 

Problem Solving Sample Question

Here’s an example of a problem solving sample question that’s retired from an actual GMAT.

On a 3-day fishing trip, 4 adults consumed food costing $60. For the same food costs per person per day, what would be the cost of food consumed by 7 adults during a 5-day fishing trip?

  1. $300
  2. $175
  3. $105
  4. $100
  5. $84

Notice how the problem solving sample question looks much more like a question that you would see in your math class or on another standardized test. All problem solving questions have five multiple choice answers.

 

Problem Solving Sample Question Explanation

The first step to solving this problem is to figure out what it’s asking you and what it requires that you know. You’re trying to figure out the total cost of food for 7 adults on a 5-day trip. In order to do that, you need to figure out the cost of food per day for each adult. You can find that by looking at the costs of the 3-day trip.

In order to determine the food costs per person per day, divide the total food cost ($60) by the total number of adults (4), and then divide that by the number of days (3). Your equation should be: (60/4)/3 = x, when x is the food cost per person per day.

(60/4)/3 = x

(15)/3 = x

5 = x

The food cost per person per day is $5. Let’s plug that into an equation to determine the cost of food on the 7-day trip: $5 (food cost per person per day) * 7 (number of adults) * 5 (number of days) = y (total cost of food on a 7-day trip)

5 * 7 * 5 = y

5 * 7 * 5 = 175

Total food cost per day on a 7-day trip is $175. So the correct answer is B.

 

10 Tips for Mastering the GMAT Quantitative Section

In this section, I’ll give you some tips that’ll help you as you practice and get ready to take the GMAT and some tips that’ll help you succeed on test day.

 

Overall GMAT Quant Study Tips

In your GMAT quantitative review, keep in mind these overall tips that’ll help you build your speed, accuracy, and confidence in solving data sufficiency and problem solving questions.

 

Master the Fundamentals

The GMAT quant section only tests high school math concepts. That means that you’ll have likely seen every skill you need to master the test. The key, then, lies in mastering these fundamentals. You won’t be able to use a calculator on the GMAT, so you’ll have to be fast with your basic calculations. Practice multiplying and dividing decimals and fractions. Memorize the exponent rules. Memorize common roots and higher powers. These simple tips will build your confidence and save you time on test day.

 

Use What You Know

GMAT quant questions are designed to look very complex and intimidating. However, no matter how difficult the question may look, remember that you’ll only need to use high school level math to answer it. Start small on these questions by using what you know. If you break the problem down into small steps, beginning with what you know, you’ll be able to work towards an answer.

 

Plug-in Smart Numbers

Plugging-in numbers is a useful strategy for solving questions. If a question gives you only variables and doesn’t ask you to solve for a number, you can pick values for the variables to make them easier to work with or to test statements.

However, you want to make sure you’re using smart numbers. -1, 1, and 2 are good numbers to plug in if you don’t have any confines, because they’re easy and manageable. If the question asks you to use a specific type of number (e.g., a multiple of 3), make sure you’re using one that’ll be easy to do basic calculations with (e.g., use 6 instead of 54).

 

Data Sufficiency Tips

The data sufficiency section is different than any other math test section you’ve seen before. Here are some tips to keep in mind as you’re preparing for and taking the GMAT.

 

Evaluate the Statements Individually First

Evaluating each statement individually will help you answer data sufficiency questions quickly and more easily. Evaluate statement 1 first, then evaluate statement 2 by itself. When you do evaluate statement 2, you’ll need to forget everything you did for statement 1. Pretend that they’re two different questions. Once you’ve determined whether each statement is sufficient on its own, you’ll be able to put them together. This strategy will also save you time. For instance, if neither statement is sufficient on its own, you’ll be able to eliminate answers A, B, and E. If both statements are sufficient on their own, you’ll be able to eliminate A, B, C, and E right away.

 

Memorize the Five Answer Choices

Every single data sufficiency questions has the same 5 possible answers:

  1. Statement (1) ALONE is sufficient, but statement (2) alone is not sufficient.
  2. Statement (2) ALONE is sufficient, but statement (1) alone is not sufficient.
  3. Both statements TOGETHER are sufficient, but NEITHER statement ALONE is sufficient.
  4. EACH statement ALONE is sufficient.
  5. Statements (1) and (2) TOGETHER are NOT sufficient.

The 12TEN mnemonic can help you remember those answers and save you time:

  • 1: only statement 1
  • 2: only statement 2
  • T: both statements together
  • E: either statement
  • N: neither statement

 

Know the Two Types of Questions

There are two basic kinds of data sufficiency questions: value questions and yes/no questions. By learning to identify which type a given question is, you can be sure you understand what it’s asking and how to answer it.

Value questions ask you to find a numerical value (e.g., what’s the value of 5x?). For value questions, if you’re able to find a specific value using the information in either statement, then that statement is sufficient.

Yes/no questions ask you whether or not something is true (e.g., is y an even number?). For yes/no questions, a definitive yes or a definitive no answer are both considered sufficient. An answer that is sometimes yes or sometimes no is not sufficient.

Remember: a definitive answer is always sufficient. An answer that may or may not be correct is not sufficient.

 

GMAT Quantitative Test Day Tips

Here are some tips that’ll help you ace the quant section on test day.

 

board-361516__480

 

Spend time reviewing the information in graphs, charts, and tables

There will be a number of questions on the GMAT quant section that require you to interpret charts, graphs, and tables. Try to spend at least 30 seconds reviewing the information on these. It’s extremely important that you read the axis labels, the key, and the units of measurement so that you’re correctly interpreting the information contained in the graph, chart, or table and using the right information to make your calculations.

 

Read questions carefully

One of the most common mistakes you can make on the GMAT is to answer the wrong question. The people who write the GMAT will purposefully include questions that ask things like “Which of the following may not be true?” which can be commonly misread or misunderstood as “Which of the following may be true?” Make sure that you read every question carefully so you’re finding the correct answer to the correct question.

 

use your scrap paper

Even though the GMAT quant section test on high school math skills, use your scrap paper as much as possible. Writing down your calculations will help you see any mistakes and force you to make sure you’re thinking through every step of the question, especially since you can’t use a calculator on the GMAT quant section.

 

work backwards

If you’re not sure where to start on a question that asks you to solve for a specific value, remember that the test has already given you the right number — you just have to find it among the wrong ones. You can work backwards by plugging in the possible answers until you find one that works.

Moreover, the GMAT normally arranges answer choices in the ascending numerical value, so you can save time by starting with the middle answer. Then you can determine whether you need to go higher or lower with your answer and eliminate multiple incorrect answers at once.

 

GMAT Quantitative Review: What You Need to Know

The GMAT quant section can be daunting, but with careful preparation and attention-to-detail on test day, you’ll be able to master it. Spend time during your practice developing a deep familiarity with the two types of questions on the quant section (data sufficiency and problem solving) and mastering the basic math fundamentals tested on this section and you’ll be well on your way to achieving your goal score.

 

What’s Next?

If you think you’ve gotten a good handle on the quant section, check out our guide on mastering the three questions of the GMAT verbal section.

Still confused by data sufficiency? Check out our in-depth guide to learn more about this unique test question type.

Find out more about what makes up your GMAT total score and how business schools weight it against your quant score.

The post GMAT Quantitative: 10 Tips to Master the Math Section appeared first on Online GMAT Prep Blog by PrepScholar.

]]>
6 GMAT Grammar Rules You Must Know /gmat/blog/gmat-grammar-rules/ Mon, 29 Jan 2018 14:00:10 +0000 http://gmat.psblogs.com/?p=2159 You may have heard rumors that you need to know a lot of grammar for the GMAT. Unfortunately, it’s true that you’ll see GMAT grammar questions; however, the test focuses on just a few main concepts you need to know. In this article, I’ll go over the six most important GMAT grammar rules, as well as … Continue reading "6 GMAT Grammar Rules You Must Know"

The post 6 GMAT Grammar Rules You Must Know appeared first on Online GMAT Prep Blog by PrepScholar.

]]>
grammar-389907_640

You may have heard rumors that you need to know a lot of grammar for the GMAT. Unfortunately, it’s true that you’ll see GMAT grammar questions; however, the test focuses on just a few main concepts you need to know.

In this article, I’ll go over the six most important GMAT grammar rules, as well as offer tips for how to learn them so you can ace the sentence correction questions on test day.

 

Overview: GMAT Grammar

Grammar is tested on the GMAT primarily in sentence correction questions, which make up about a third of the verbal section (36 questions in total).

In sentence correction questions, you will be shown a sentence with an underlined portion, and five answer choices that give options for how to replace the underlined portion. The first answer choice is always exactly the same as the underlined portion of the sentence.

Sentence correction questions on the GMAT will ask you to select the answer choice that will make the sentence 1) grammatically correct, 2) concise, and 3) logical. Most questions will test you on more than one of these factors at a time.

Keep in mind as you prepare for this section of the GMAT that GMAT grammar is based on specific rules. Even if a sentence would sound correct in normal daily conversation or writing, it might not be correct by GMAT standards, so it’s important to learn each of the grammar rules tested by the exam.

The good news is that GMAT grammar rules are also repetitive: as you practice, you’ll see the same rules being tested over and over again, which means that if you prepare well and thoroughly, you’ll know exactly what to expect on sentence correction questions.

 

Grammar is tested primarily on sentence corrections questions on the GMAT.
Grammar is tested primarily on sentence corrections questions on the GMAT.

 

6 Key GMAT Sentence Correction Rules

Let’s go over six of the most common GMAT grammar rules, alongside examples (authored by the Graduate Management Admissions Council, GMAC, which also writes the GMAT itself) from the GMATPrep Software.

 

#1: Dangling Modifiers

The GMAT loves to test you on modifiers, which are clauses, words, or phrases that describe other parts of a sentence. In the sentence, “My sister, the psychologist, is coming to visit,” for example, “the psychologist” modifies “my sister,” which is the subject of the sentence.

Dangling modifiers are the most common modifier-related errors on the GMAT. In a dangling modifier, the modifying phrase is misplaced, describing the incorrect word or phrase so that the sentence is illogical. Take a look at the following sentence and see if you can tell why it’s illogical.

A beautiful red-haired mermaid, the president thought Ariel was highly intelligent.

This sentence contains a dangling modifier: The president is likely not a beautiful red-haired mermaid (though that would be cool). The modifier “A beautiful red-haired mermaid” belongs next to the noun it modifies, which is Ariel. Let’s rewrite it to remove the dangling modifier:

The president thought Ariel, a beautiful red-haired mermaid, was highly intelligent.

Now let’s take a look at an official GMAT example.

dangling modifier 2

To spot dangling modifiers, look for introductory phrases followed by a comma. In this question, “architects and stonemasons” is the modifying phrase in question. The phrase needs to modify a noun, and one that is logical in the context of the sentence.

In answer choices B, D, and E, the modifying phrase is not following by a noun or noun phrase (answer choice E begins with a verb, ‘were,’ for example), so you can eliminate them.

In answer choice A, the phrase “architects and stonemasons” is modifying “huge palace and temple clusters.” It’s not logical for the palace and temple clusters to be architects and stonemasons, so you can eliminate this answer choice.

In answer choice C, “architects and stonemasons” is modifying “the Maya,” which makes sense in the context of the sentence. This is the correct answer.

 

#2: Proper Verb Tense

GMAT sentence correction questions often test students on proper verb tense usage. Remember that verb tenses (the basic ones are past, present, and future) give us information about when an action took place (or will take place, etc.). Verb tenses are generally supposed to remain consistent, unless there is a specific reason for the tense to shift within a sentence. For example:

By the time the police arrived, the robbers had fled.

Note the clue that the verb tense is supposed to change (in this case from past tense to past perfect): “By the time” lets us know that there was a sequence of events. In this sentence, the robbers fled before the police arrived, so that shift in time needs to be reflect by a shift in verb tense.

In the GMAT example below, the conditional construction of “if x happens, y will happen” requires the first half of the sentence to be in present tense and the second half of the sentence to be in the future tense. So we can immediately eliminate answer choices A and B.

verb tense shifts

Answer choices C and D are less concise and precise than answer choice E because of the extraneous clauses they include (‘as they already did’) and (‘as they have already’), making answer choice E the correct option.

 

#3: Illogical Comparison

In the sentence “Mark loves chips more than his mom,” what’s wrong?

Well, unless Mark really loves chips, it’s unlikely to be true. What the writer is probably trying to say is “Mark loves chips more than his mom does.” The original sentence is an example of an illogical comparison.

Illogical comparisons compare apples to oranges, meaning that the things being compared either don’t make sense in context or don’t match up with each other grammatically (nouns being compared to verbs, for example).

Clues of an illogical comparison question on the GMAT include phrasing like “more/less than,” “like/unlike,” and “as…as.”

In this example, you’re being asked to compare “a typical automobile loan” to something else (the word “unlike” clues you into this). So you need to look for 1) a noun, since “ban” is also a noun, and 2) a noun that’s logical in the sentence’s context.

illogical comparison 5

Answer choices B and D can’t be correct, then, because they compare the “automobile loan” (a noun) to prepositions (“with” and “for”).

Answer choices A and C are incorrect because, while they start with nouns (“lease-loan buyer(s)”), they are illogical; a lease-loan buyer can’t be compared to a loan itself.

Answer choice E is correct. It compares “a typical automobile loan” to “a lease-loan.”  

 

Illogical comparisons can be spotted using clue words like "like" and "unlike."
Illogical comparisons can be spotted using clue words like “like” and “unlike.”

 

 

#4: Pronoun Usage

GMAT sentence correction questions test you on proper pronoun usage. Correct pronouns match their antecedent (the word they stand in for). For example, in the sentence, “Sara took off her hat,” “Sara” is the antecedent for “her.” Plural pronouns (we, they, them, us) match with plural antecedents, and singular pronouns (I, he, she, her, him, it) match with singular antecedents.

Let’s look at one example of incorrect pronoun usage. What’s wrong with this sentence?

I never go to that restaurant because they have moldy cheese.

Who has moldy cheese? We can’t tell from the sentence. “They” is a plural pronoun, and its possible antecedents (“I” and “restaurant”) are both singular. We would rewrite it like this:

I never go to that restaurant because it has moldy cheese.

Now let’s take a look at a GMAT example of incorrect pronoun usage.

pronoun usage 2

In this example, the antecedent of “it” is “crab.” “They” and “their” (both plural pronouns) are incorrect pronouns because they don’t match with their antecedent (singular), so you can eliminate answer choices B and E. Answer choices A and D have no pronoun, so they are illogical (“because of living at great depths”—who/what is living at great depths?).

Answer choice C includes the correct, singular pronoun (“it”) and the appropriate verb (“lives”), so it’s the right option. Notice that it is also more concise than answer choices A and B, which is often a clue that an answer choice is a good one.

 

#5: Subject-Verb Agreement

GMAT sentence correction questions often include subject-verb agreement errors. The subject of a sentence must match with its verb; a singular subject has to accompany a singular verb, and a plural subject has to accompany a verb in the plural form.

A common trap you’ll see in subject-verb agreement questions is a prepositional phrase (a phrase that starts with a preposition and ends with a noun) that is wedged between the subject and the verb to confuse you.

Let’s look at an example of the “distracting extra phrase” trap:

This box of decorations belong upstairs.

The prepositional phrase in this sentence is “of decorations,” which you should ignore when figuring out the correct verb form to accompany the subject. “Box” is the subject, and it’s singular, so the verb form should be too; the singular present tense form of the verb “to belong” is “belongs,” not “belong,” so the sentence should read, “This box of decorations belongs upstairs.”

Now let’s go through a GMAT example of a subject-verb agreement error.

subject verb agreement 2

The subject you’re dealing with in this example is “cost” or “costs.” Its verb is “are” (before “prohibitive”), which is plural, so the subject has to be plural as well. This allows you to eliminate choices A and D right off the bat.

We’ll go over how to select the correct answer choice for this question in the next example.

 

#6: Idioms

An idiom, or idiomatic expression, is a common phrase in English, often involving prepositions. For example, one graduates “from” college, not “of” college. One “belongs to a club,” not “belongs with a club.”

GMAT sentence correction questions generally test more than one grammar error at once, and you’ll often see wrong answer choices that fix the original error but contain an incorrect idiom. Correct answer choices will always be grammatically and idiomatically sound.

Let’s look at an example:

idioms9

When two choices are being compared, “whether” is the idiomatically proper choice rather than “if.” That means that answer choices A and D can be eliminated immediately based on their incorrect use of “if.”

“Being” is redundant (and unnecessarily wordy) when used after “undergoing,” because “undergoing” already lets you know that the conversion is in process, so answer choices B and C can be eliminated as well.

Answer choice E is correct based on idiom usage, a lack of redundancy, and concision.

 

3 Tips for Learning GMAT Grammar Rules

There are a few key ways to target GMAT grammar to prep for the sentence correction questions. Let’s go over the three top tips for mastering the exam’s grammar questions, undertaking effective GMAT grammar practice, and learning GMAT sentence correction rules.

 

#1: Use Flashcards to Study Idioms

It’s important to learn about the most common idioms you’ll encounter on the test as part of your GMAT grammar practice routine so you can eliminate answer choices that use idioms incorrectly right away. Once you get familiar with them, answer options with incorrect idiomatic expressions speed up the process of elimination.

Particularly if you’re a non-native English speaker, learning idioms can be a tricky aspect of your GMAT grammar prep. Creating and studying with flashcards that include the meanings of idioms as well as examples of their proper usage can help you get comfortable with GMAT idioms. Check out our guide to GMAT idioms to familiarize yourself with the most common ones.

 

#2: Review the Underlying Grammar Concepts

Your GMAT grammar practice should include a review of the basics of grammar, including parts of speech, sentence structure, modifiers, verb tense, and pronoun usage. Princeton Review’s Grammar Smart includes an overview of many of the GMAT-relevant grammar rules, as well as drills to complete as part of your prep. I would recommend Grammar Smart as your primary GMAT grammar book over generic grammar guides because they aren’t likely to be specifically relevant to the GMAT.

 

#3: Learn How Grammar Rules Are Tested on the GMAT

When you practice with the GMATPrep Software or other study resources, target GMAT grammar rules by selecting sentence correction practice questions. As you learn to identify various question types, you’ll become more aware with the way each of the GMAT sentence correction rules are tested on the exam. This will help you learn how to approach each grammar concept as it is presented on the GMAT and how to work through each question type more efficiently, effectively, and accurately.

 

Regular practice is key to learning the GMAT grammar rules.
Regular practice is key to learning the GMAT grammar rules.

 

What’s Next?

For more information about the verbal section as a whole, check out our article on mastering all three GMAT verbal question types.

Our guide to the best GMAT verbal practice materials will help you prepare for the verbal section of the exam more effectively.

Trying to understand each of the GMAT sections in more depth? Our comprehensive overview of all four sections of the test will help you out.

The post 6 GMAT Grammar Rules You Must Know appeared first on Online GMAT Prep Blog by PrepScholar.

]]>
2000+ GMAT Sample Questions: Practice Every Question Type /gmat/blog/gmat-sample-questions/ Mon, 04 Dec 2017 17:00:39 +0000 http://gmat.psblogs.com/?p=1300 Practice questions are an essential part of any GMAT prep. But how do you find the best GMAT sample questions? In this article, I’ll address what to look for in GMAT practice questions, the pros and cons of both official and unofficial GMAT test questions, and tips on creating an effective GMAT study plan using these … Continue reading "2000+ GMAT Sample Questions: Practice Every Question Type"

The post 2000+ GMAT Sample Questions: Practice Every Question Type appeared first on Online GMAT Prep Blog by PrepScholar.

]]>
woman-792162_1280

Practice questions are an essential part of any GMAT prep. But how do you find the best GMAT sample questions?

In this article, I’ll address what to look for in GMAT practice questions, the pros and cons of both official and unofficial GMAT test questions, and tips on creating an effective GMAT study plan using these resources.

 

What to Look for in GMAT Sample Questions

There are a wide variety of GMAT sample questions out there. How do you know if you’re using quality practice ones? These are a few of the characteristics of good GMAT practice questions, whether they’re official or unofficial.

 

#1: Same Format and Question Types as the Real GMAT

Once you start taking GMAT practice tests, you’ll get a feel for how GMAT questions should look: how long they are, what they look like on the screen, the topics they tend to cover, and the style in which they’re written, for example. In every section, there will also be a mix of question types: the quant section always includes both data sufficiency and problem-solving questions, while the verbal section always includes a mix of sentence correction, reading comprehension, and critical reasoning questions.

The GMAT example questions you choose as you prepare for the test should be as similar as possible to real questions written by the Graduate Management Admissions Council (GMAC). The closer you can get to simulating real testing conditions, including everything from visuals to question content, the more comfortable you’ll feel on the day of the test. Also, answering GMAT practice questions that are similar to those you’ll encounter on exam day will help you gauge your progress and potential GMAT score range accurately.

 

#2: Computerized (and Hopefully Adaptive)

The GMAT is a computerized adaptive test (CAT), which means it adapts to the test taker’s skill level in real time to determine both their score and the mix of questions they’re given. Ideally, GMAT sample questions should follow this format, so you can be best prepared for what you’ll see and experience on test day. Practice resources in CAT format will usually advertise that fact prominently. If they don’t, they’re likely not in the format of the official GMAT.

Obviously not every resource you use will be in CAT format—for example, if you’re using a book to do drills on specific question types. But a sizable portion of your prep should be done using practice questions in CAT format.

 

#3: Accurate Difficulty Levels

The GMAT always includes a mix of easy, medium and difficult questions. How many you receive of each will depend partly on the computerized adaptive test: You’ll start with ‘medium’ questions, and if you answer those correctly, you’ll be given more difficult questions on average; if you answer them incorrectly, you’ll get easier questions on average. However, every test taker will receive some questions at all three difficulty levels.

Your selection of practice GMAT questions should be at the same difficulty level as the real GMAT test questions. As you prepare, you’ll be able to tell if your sample GMAT questions seem too easy or too difficult on average. Practicing with either too-difficult or too-easy questions might give you an inaccurate picture of your probable ultimate GMAT score, and could also leave you underprepared for the real GMAT.

 

#4: Organized by Skill and/or Difficulty

You should target your weaknesses when completing GMAT practice questions. You may struggle more with data sufficiency questions than with problem-solving questions on the quant section, for example, or you may do well on most geometry questions but wrestle more with trigonometry and algebra. Alternatively, you may have mastered easy questions across the board and might need to focus on medium and/or difficult questions instead.

Resources that offer customizable sets (based on question type, skills tested, and difficulty level) of GMAT practice questions, or that separate the questions according to those categories, will help you study more effectively and in a more organized fashion than ones that aren’t labeled according to type or that are randomly compiled. The more specifically each question is categorized, the more effectively you will be able to form a study plan that hones in on your specific problem areas.

 

Computerized adaptive practice questions will help you prepare most effectively for the GMAT.
Computerized adaptive practice questions will help you prepare most effectively for the GMAT.

 

Official GMAT Practice Questions

The Graduate Management Admissions Council (GMAC) releases a variety of practice resources that use official GMAT sample questions, either written by the test makers themselves or adopted from retired exams. For each resource, I’ll go over what’s included, its strengths and weaknesses, and the best way to make use of it in your prep.

 

#1: GMATPrep Software

The official GMATPrep Software is the best resource for GMAT practice questions available. It contains two full-length simulated computerized adaptive tests. Everything is written by GMAC, so you know you’re getting the real thing. You can take the two practice tests as many times as you like, but you’ll likely repeat some of the same questions if you take each one more than once.

You can customize your sets of practice questions, choosing 1-15 questions of each type (critical reasoning, sentence correction, and reading comprehension, for verbal) and difficulty (easy, medium, or difficult). In addition to the questions on the two practice tests, there are 90 additional practice questions available on the software.

The software also includes a detailed breakdown of GMAT question types and strategies, as well as a review of skills you’ll need for the quant section.

The Good:

  • It’s free for registered users of mba.com. Just create an account.
  • The practice questions are identical in length, format, style, content, and visuals to what you’ll see on the GMAT.
  • Answer explanations are step-by-step and in-depth, and you can go back and review or drill questions you previously got wrong as many times as you like.
  • Detailed performance reviews break down what you’re getting wrong so you can hone in on your weak spots, including specific question types and skill sets

The Bad:

  • Not much! The GMATPrep exams are great GMAT example tests. There aren’t many resources that can beat official questions under simulated testing conditions, and it’s free.
  • However, there are only two full-length practice tests, so you’ll likely need to supplement these practice questions with other resources.
  • Also, while the quant review is great, there isn’t a similar resource available on the software for the verbal section.

How to Use It:

  • It’s best to start off your GMAT prep by taking one of the two full-length simulated tests on the GMATPrep software. Since the practice questions and score report are so realistic, the results will serve as a good barometer of your starting point.
  • After you complete the rest of your prep with other practice questions, you should take the second GMATPrep practice test a few weeks before you take the GMAT. This will give you a good idea of where you stand before exam day.

 

#2: GMATPrep Question Pack 1

An official addition to the GMATPrep Software, the GMATPrep Question Pack contains 404 additional official questions.

You can create custom sets of practice questions of any length based on your needs. Each set of questions can be completed in either ‘exam mode’ (timed, under simulated test conditions) or ‘study mode’ (untimed). The Question Pack will cost you $29.99 to download.

The Good:

  • This resource contains practice questions that aren’t available in any other official prep materials.
  • Your performance reviews for these practice questions will be integrated into your overall progress reports in the GMATPrep Software, allowing you to get an even more accurate picture of what your needs are in terms of prep.

The Bad:

  • There isn’t much of a downside to official questions as a supplement to your overall prep.

How to Use It:

  • Once you’ve taken a few practice tests and figured out your weaknesses, create custom practice question sets that target your difficult spots (particular sections, question types, or question difficulty levels). These will serve as drills to beef up your skills in those areas.

 

GMAT customizable question banks allow you to work on your timing.
GMAT customizable question banks allow you to work on your timing.

 

#3: The GMATPrep Exam Collection

These are official additions to the GMATPrep Software. You can only get them once you’ve downloaded the original free software.

Each Exam Pack contains 90 additional practice questions (30 quantitative, 45 verbal, and 15 integrated reasoning), as well as two full-length computerized adaptive GMATs. Each Exam Pack costs $49.99.

The Good:

  • In-depth diagnostics will let you know how you’re doing in comparison to your peers on every subsection of the test, as well as how your pacing could improve on each question type.
  • Answer explanations offer step-by-step, detailed analyses of all practice questions.

The Bad:

  • Not much (official GMAC questions are always helpful), except that they’re a bit pricey.

How to Use It:

  • These practice questions are a great first step after you take an initial diagnostic practice test, particularly if you’re still unsure of what exactly your weak spots are.
  • You can use the two full-length CAT GMATs to gauge your progress throughout your prep. Alongside the official GMATPrep Software, you’ll have four full-length tests, so you can space them out to regularly check up on how your score is improving.

 

#4: GMAT Write

GMAT Write is a fairly new tool released by the makers of the GMAT, meant to help you with the analytical writing assessment. It includes two unique sample prompts for the analytical writing assessment section.

GMAT Write will time your essays (30 minutes each, just like on the real exam) and score them in real time according to the GMAC rubric. It costs $29.99 to download.

The Good:

  • The visuals and experience of writing the essay are highly realistic.
  • The sample questions are written by GMAC, so you know they’re reliable.
  • GMAT Write scores your essays according to the same criteria used on the real GMAT— ‘analyzes the issue,’ ‘supports ideas,’ ‘organizes a coherent idea,’ and ‘language control’— in real time. It’s the only official GMAC tool that will score your sample essays.

The Bad:

  • There are only two unique exam prompts included, which isn’t necessarily a lot of practice for $29.99.
  • You won’t get any detailed feedback, just scores in each of the four categories used to assess GMAT essays.

How to Use It:

  • Use it alongside other practice tests. Many other resources only have integrated reasoning, verbal and quantitative practice questions, so adding a prompt from GMAT Write to your practice test will help you more closely simulated the experience of the actual GMAT.

 

GMAT Write will help you prep for the Analytical Writing Assessment.
GMAT Write will help you prep for the Analytical Writing Assessment.

 

#5: The GMAT Official Guide 2018

The GMAT Official Guide 2018 is a comprehensive resource for GMAT prep. Included in your purchase ($19.95) is access (both online and in print) to over 900 official practice questions, access to an accompanying site where you can customize sets of practice questions, and online videos with plenty of GMAT tips and strategies.

The Good:

  • The questions are written by GMAC, so they’re high quality. The online practice questions mimic the visuals of the real GMAT.
  • The practice questions are organized in order of difficulty.

The Bad:

  • The answer explanations and introduction sections (where key GMAT concepts get broken down) are too complex for non-advanced students. This is especially true for the quant and sentence correction questions.
  • The online question bank only allows you to save ten practice sessions at a time, so if you plan on doing more, you’ll need to delete them before starting a new session.

How to Use It:

  • The online question bank is great for regular practice sessions. The question sets are customizable, and you can review them at any time.
  • The book itself is a good resource to start your practice with after using the GMATPrep software as an initial diagnostic tool.

 

#6: IR Prep Tool

The Integrated Reasoning (IR) Prep Tool was introduced in 2014 to aid students specifically in the integrated reasoning section.

The tool includes 48 integrated reasoning questions total: 10 graphics interpretation questions, 15 multi-source reasoning questions, 15 two-part analysis questions, and eight table analysis questions.

You can create customized practice question sets, focusing on all question types or just one. You can also set the difficulty level for each practice question set. Once downloaded, you have unlimited use of the prep tool for six months. The IR Prep Tool can be downloaded for $19.99.

The Good:

  • A detailed ‘help’ section breaks down each integrated reasoning question type for you and gives tips on how to approach it.
  • A time management tool tracks your average time on each question type and difficulty level, allowing you to work specifically on your pacing.

The Bad:

  • The IR Prep Tool doesn’t give you a specific score estimation. Your official IR score will be between 1 and 8, but the IR Prep Tool gives you a score between 0 and 100 based on your performance and time management.

How to Use It:

  • There aren’t many resources out there specifically aimed at the integrated reasoning section, a frequent problem area for students, so this is a goldmine for students who have trouble with it.
  • Use it to target and break down the integrated reasoning section into manageable chunks, particularly if you get overwhelmed by the fact that each question has multiple components.

 

#7: GMAT Focus Quantitative Diagnostic Tool

The GMAT Focus Quantitative Diagnostic Tool consists of a 24-question quantitative test (12 data sufficiency questions and 12 problem-solving questions) that uses real questions from retired exams. It’s computer adaptive and follows the style and format of the actual GMAT quant section.

You can start and stop the test manually, but you have 45 minutes total to complete the practice questions.

When you complete the test, the Focus Tool provides detailed answer explanations and an assessment of your strengths and weaknesses in terms of quant concepts.

Four unique tests are available. One test costs $29.99, and a three-test package costs $79.99.

The Good:

  • The GMAT Paper Tests are the only other GMAT practice resources that use real retired GMAT questions, making the Focus Tool the only computerized adaptive resource to do so. So you get the best of both worlds with this one: real practice questions from previous GMATs and simulated testing conditions.
  • The detailed assessment of your performance, including a possible score range (more accurate than an exact score, since no one practice test will give you a perfect prediction of your ultimate GMAT score) and a breakdown of your performance by question difficulty, will tell you exactly which quant skills you need to work on before the GMAT.

The Bad:

  • It’s a bit pricey for just 24 questions each.
  • It’s not tied into the other GMAT prep resources, so your results won’t be integrated into your GMATPrep performance reports.

How to Use It:

  • After you complete one of the GMATPrep exams, this tool will help you hone in on the quant section. You’ll get an overall idea of your strengths and weaknesses in quant from the GMATPrep Software, but this tool will help you get much more specific about which skills to target in your quant prep.

 

The Quantitative Focus Diagnostic Tool will help you hone in on the intricacies of the GMAT quant section.
The Quantitative Focus Diagnostic Tool will help you hone in on the intricacies of the GMAT quant section.

 

#8: GMAT Paper Tests

The GMAT Paper Tests are real retired GMAT tests, written by GMAC. Each set (I, II, and III) contains three official GMAT tests, answer sheets, and a guide for converting your raw score into a scaled score. They’re available in downloadable PDF form. Each set costs $29.99.

The Good:

  • Most of the questions in these tests aren’t available in any of the other official GMAT practice tests or resources, so they’re especially good if you want some extra practice. This is a big plus: It’s somewhat difficult to find GMAC-authored questions that don’t overlap with other resources.
  • Official questions are always a big positive in terms of GMAT prep!

The Bad:

  • They’re on paper! You’ll have to score them yourself using the answer sheet, and it’s not computerized (obviously) or adaptive, so it doesn’t simulate any of the testing conditions.
  • No answer explanations are included, just the answers themselves.

How to Use It:

  • You may want to order these if you’re a test-taker with a disability that may require an accommodation and using paper tests at home is easier for you during prep time.
  • The GMAT Paper Tests might also serve you well if you’ve used all the other official sample GMAT questions and you want some additional prep. However, don’t rely on them to give you much info on your likely final GMAT score, since they aren’t in the official GMAT format. Use them to work through practice questions in your prep sessions instead.

 

Best Unofficial GMAT Practice Questions

Official GMAT questions are great, but they aren’t the only GMAT prep materials out there. There are also a variety of free and paid resources that offer quality GMAT practice questions. Here, I’ve included practice tests, books, and online question banks. For each unofficial resource, I’ll address how you can access it, its pros and cons, and the best way to incorporate it into your GMAT prep.

 

#1: Veritas Prep

You’ll need to create an account to access Veritas Prep’s GMAT Question Bank, but once you do, it’s completely free.

The Question Bank offers hundreds of realistic sample GMAT questions: quant, verbal, and integrated reasoning. It includes example questions of every type within those categories. Before you start your practice session, you can choose which kinds of questions you want to see. With one account, you’ll only see each practice question once.

Veritas Prep doesn’t give you an official estimated score, but it does give you an overall percentile, which measures how you did on a particular question set compared to other students who answered the same practice questions.

The Veritas Prep Question Bank is ideal for regular practice throughout your prep, particularly if you have varying amounts of time (you can do a short or lengthy session) and want to drill certain skill sets or question types. The questions from the bank aren’t in CAT format, but you can access one free computerized adaptive test through Veritas and six paid ones. Veritas Prep questions are some of the closest you’ll find to official GMAC-authored GMAT example questions.

 

#2: Kaplan

Kaplan has a wide variety of GMAT resources, including in-person and online prep courses, but its GMAT Premier 2018 is its flagship GMAT prep tool. Your purchase includes access to over 1,000 GMAT practice questions, six full-length practice tests (one is in the book, and five are computerized adaptive tests available online), video tutorials, an online question bank with 200 GMAT sample questions and customizable quizzes, and a mobile app that allows you to study on the go.

GMAT Premier also includes a special section of forty advanced quant practice questions and in-depth answer explanations, which can help if you’re looking for a challenge in the quant department. The Kaplan practice questions are a bit harder than the ones in Princeton Review, but the verbal questions in particular still leave something to be desired in terms of difficulty level. The sentence correction questions in particular are easier than real GMAT questions. The Kaplan quant questions fare a bit better in terms of GMAT comparability.

Like Princeton Review, Kaplan is a good resource if you’re looking for a fairly solid overview and breakdown of the exam to start your prep (after you take your first diagnostic GMATPrep test). It’s not going to be as helpful if you’re looking for a comprehensive guide to the GMAT or a massive boost in your score.

 

#3: Manhattan Prep

With an account, you can take one full-length computerized adaptive Manhattan Prep test for free. Six additional ones are available for $49.99, which gives you one year of unlimited access.

You can take the Manhattan Prep practice GMATs timed or untimed, and you can take complete or partial practice exams depending on your time limit during prep. After you take all six, you can reset the questions to take more. Each test includes an in-depth assessment of your performance that analyzes your strengths, weaknesses, timing, and the difficulty levels you’ve mastered.

Manhattan Prep questions are mostly quite realistic in content and difficulty level. Though the visuals aren’t perfect, they are not too far from what you’ll see on the real exam.

I’d also like to point out one of Manhattan Prep’s best resources for students looking to challenge themselves in the quant section: Their GMAT Advanced Quant book includes strategies for tackling the trickiest quant questions and over 150 realistic practice questions. This is a great resource if you’re already doing well in the quant section and looking to bump yourself up to the next level.

 

#4: PowerScore

PowerScore’s main claim to fame is its trilogy of GMAT Verbal Bibles: Critical Reasoning, Sentence Correction, and Reading Comprehension, along with the PowerScore Verbal Bible, which covers the entire section. The books are available for $24.99 each or $69.99 as a trilogy.

PowerScore does an excellent job of breaking down each question type within the verbal section (a great resource for non-native English speakers and anyone who struggles with verbal questions), and the sample GMAT questions included in each book are consistent with the length, difficulty, and content of those on the GMAT. Though of course the practice questions here aren’t computerized or adaptive, each book does include access to a companion website that includes extra drills and practice questions.

The main downside to PowerScore is that, while the books offer great strategies and explanations of various question types, there aren’t a substantial number of actual practice questions. The Verbal Bible, for example, contains 31 critical reasoning questions, 77 sentence correction questions, and only eight reading comprehension questions. So while these resources are great reads for test-takers who are looking for in-depth answer explanations, they’re not fantastic for substantial extra practice or regular prep sessions.

 

The PowerScore Verbal Bibles have excellent grammar guides to help you with sentence correction questions in particular.
The PowerScore Verbal Bibles have excellent grammar guides to help you with sentence correction questions in particular.

 

#5: 800 Score Test

Providing a username and password will get you one full-length test on 800 Score Test for free. You can download five more for $39.95, which includes a comprehensive quant and verbal review, ten sample essay questions, access to online help, and in-depth video explanations of practice questions.

The visuals at 800 Score Test aren’t much like what you’ll see on the GMAT, but the practice questions themselves are high quality, particularly in the quant section. The verbal practice questions veer a little ‘off’ and aren’t as close to the real GMAT verbal section as the quant questions are, so if you’re especially looking to focus on verbal practice questions, this might not be your best bet.

The 800 Score tests are in CAT format, though, so even though the visuals are a bit lacking, they’re not the worst resource if you only want practice questions that simulate real testing conditions. Their Test Pacer tool lets you know what question you should be on to finish in time, which is very helpful for students who have trouble with consistent pacing.

 

#6: Princeton Review

Princeton Review’s Cracking the GMAT, available for $21.99 on Kindle, offers a breakdown of all GMAT sections (including integrated reasoning and the analytical writing assessment), along with six full-length computerized adaptive practice exams, over 180 practice questions organized by difficulty level, and drills for each test section. Cracking the GMAT Premier is $7 more and comes with more online resources, like extra practice tests, study strategies, and video tutorials.

Princeton Review offers a decent overview of each of the GMAT sections, though it only offers the grammar and math fundamentals, not anything in the way of more advanced concepts. Indeed, where it misses the mark is mainly in the difficulty of the practice questions: Nearly none of them are as difficult as actual GMAT questions. This is especially true in the critical reasoning and reading comprehension sections. So while Cracking the GMAT may help beginners who aren’t familiar with GMAT basics (especially to start off, since it’s not an intimidating text), it’s not enough for test-takers looking to break 700 or even 650.

 

#7: GMAT Club

Beyond its helpful GMAT forums, GMAT Club offers a variety of useful GMAT practice materials, including practice tests and question banks.

A few practice tests and question banks are free (no account required), but access to all of their customizable quizzes, bonus questions (over 1,572) and full-length tests will run you at least $79.99 for three months of use.

GMAT Club tests aren’t exactly in CAT format, as they don’t adapt to your performance as you go along, but they do follow CAT ‘logic’ and include a mixture of difficulty levels and question types that is consistent with what you’ll usually encounter in a computerized adaptive test. The quality of the visuals and practice question content is high, with practice questions closely resembling GMAC ones.

Difficult quant questions are one of GMAT Club’s specialties, which is important as many of the other top practice resources (Princeton Review and Kaplan, for example) trend a little easier: They claim that hundreds of their quant questions are at the 700+ scoring level. So if you’re looking to wow your prospective schools with your quant score, this might be a great resource.

 

#8: Manhattan Review

You have several options for free practice questions and other GMAT prep materials with Manhattan Review. With an account, you can access a fairly high-quality quantitative question bank in PDF ebook form, along with a vocabulary list (good for non-native English speakers) and another free ebook, GMAT in a Nutshell, that serves as an overall guide to the test and top strategies.

The Sentence Correction Guide, downloadable for free, boasts a solid grammar review of the concepts that are more often tested in sentence correction questions, but the sentence correction practice questions themselves aren’t all quite as difficult or complex as the real ones on the GMAT.

Manhattan Review also offers a full-length free practice test, which you can access with an account. It’s a computerized adaptive test. The questions here are good for extra practice, but the difficulty levels and visuals aren’t always consistent with the actual GMAT.

 

Vocabulary practice is an integral part of GMAT prep for non-native English speakers.
Vocabulary practice is an integral part of GMAT prep for non-native English speakers.

 

3 Top Tips for Creating a GMAT Study Plan

Once you’ve selected some practice GMAT questions, it’s time to set up a study plan. It’s important not just to find the best prep materials for you, but to use them to your advantage. Here are a few tips for using your prep materials as effectively as possible:

 

#1: Use the Two Official GMAC Tests on the GMATPrep Software First and Last

You should start your prep with one of the two full-length GMATPrep Software tests. Since they’re computerized adaptive tests with accurate visuals and questions written by GMAC, they’re the best possible indicator of your ultimate GMAT score. The first practice test will give you a good idea of where you stand and how to begin your prep. Unless you purchase more official GMAC practice tests, take the second free test from the software a few weeks before your exam date. This will give you a good idea of your score at the end of it all, and will let you know what to brush up on last-minute before the big day.

 

#2: Use Different Materials to Build Different Skills

Tailor the practice GMAT materials you select to your particular needs. For example, if you’re strong in quant but weaker in verbal (or a non-native English speaker), you might use Manhattan Prep, the GMAT Focus Quantitative Diagnostic Tool, or GMAT Club practice questions for your quant prep, since they all feature more advanced quant questions. You might start out more slowly in verbal, completing grammar reviews and drills from one of the PowerScore Bibles before delving more into practice questions. Mix and match resources according to your personal GMAT goals.

 

#3: Note Differences Between the Conditions of Your Practice and the Conditions of the Real GMAT

Not all of your GMAT practice questions need to be (or should be) in computerized adaptive form. Sometimes you’ll need to take your time to work through a particular question, complete a drill, or review concepts. But when you do take a full-length GMAT practice test, stick to the real testing conditions as much as you can: Use double-sided scratch paper, don’t use a calculator on the quant section, take the right amount of time for each section, and take eight-minute breaks between the integrated reasoning and quant sections and before the verbal section.

Some GMAT practice materials allow you to manually time yourself, take longer or unlimited breaks, or stop and start the test. Be mindful of these differences so you don’t learn to rely on conditions (such as a longer break) that won’t exist on the day of your exam.

 

whats-next-1462747_1280

 

What’s Next?

Looking for free full-length practice tests in addition to the best practice questions? You can find a list in our guide to the best free GMAT practice tests.

Our complete collection of GMAT practice tests provides a more comprehensive overview, in addition to reviews, of all available GMAT practice tests.

Finally our guide to how long to study for the GMAT will help you set up a time-effective study plan.

The post 2000+ GMAT Sample Questions: Practice Every Question Type appeared first on Online GMAT Prep Blog by PrepScholar.

]]>
The 11 Best GMAT Reading Comprehension Strategies /gmat/blog/gmat-reading-comprehension-strategies-tips/ Mon, 20 Nov 2017 17:00:04 +0000 http://gmat.psblogs.com/?p=1751 The reading comprehension part of the GMAT Verbal section can be intimidating at first. You have to glean key ideas and information from dense, jargon-filled passages, which probably don’t resemble the kinds of articles you read for fun. Luckily, whether you love these kinds of passage-based questions or they fill your heart with dread, there are a … Continue reading "The 11 Best GMAT Reading Comprehension Strategies"

The post The 11 Best GMAT Reading Comprehension Strategies appeared first on Online GMAT Prep Blog by PrepScholar.

]]>
Feature_bookandglasses

The reading comprehension part of the GMAT Verbal section can be intimidating at first. You have to glean key ideas and information from dense, jargon-filled passages, which probably don’t resemble the kinds of articles you read for fun.

Luckily, whether you love these kinds of passage-based questions or they fill your heart with dread, there are a number of tried-and-true strategies that you can use to improve your score. In this post, we’ll give you all the tips and tricks you need to ace GMAT reading comprehension—including how to approach the passages overall, how to attack the questions, and how to pace yourself so that you spend just the right amount of time on them.

 

GMAT Reading Comprehension Overview

You always have 36 questions in total on the GMAT Verbal Section. Of those, you’ll encounter about three or four reading comprehension passages, each with three to four multiple-choice questions associated with it. So that makes for 9-16 reading comprehension questions, or roughly one-third of the GMAT Verbal section devoted to reading comprehension.

Within that, there will usually be three “short” passages of 200-250 words with three questions, and one “long” passage of 300-350 words with four questions. Occasionally, you might get two of each. The passages are sourced from textbooks, journals, periodicals, scholarly articles and the like, and they center on topics from the natural and social sciences, the humanities, and the business world. Note that no outside knowledge is expected in any of these domains—all the information you will need to answer the questions is in the passages themselves.

The GMAT uses a split-screen presentation for reading comprehension questions: the passage is presented in its entirety on the left side of the screen, while the questions associated with it are presented one-by-one on the right side. This means that you can always see the passage, but you can only see one question at a time. Plus, as with every other section and question type on the GMAT, you cannot go back to the previous question—your answer, once you click “next,” is final.

The reading comprehension questions on the GMAT are designed to test your ability to “understand, analyze, and apply information and concepts presented in written form”. Within this, you’ll need to:

  • Understand words and statements
  • Identify logical relationships between ideas and concepts
  • Draw inferences
  • Point out flaws in assumption and reasoning
  • Understand and follow quant concepts as presented in writing

The GMAT uses a few distinct question styles to test these different skills, all of which we discuss in the next section!

 

Reading comprehension passages can seem a bit maze-like.
Reading comprehension passages can seem a bit maze-like.

 

GMAT Reading Comprehension Question Types

Reading comprehension questions on the GMAT focus on your ability to understand the given passage on either a micro/”little picture” level or macro/”big picture” level. Accordingly, all the reading comprehension questions you’ll encounter on the GMAT will fall into one of the following categories.

 

Main Idea

Main idea questions ask you to summarize the passage as a whole or to identify the author’s overall purpose in writing the passage. You might see questions asking you to pick out the “central” or “main” idea or asking you what the author was “primarily concerned with.”

Often, the answer choices given don’t use exactly the same language as the passage itself: they want to see that you really understand the main idea, as evidenced by being able to recognize it even when it’s phrased differently. They especially want to see that you can recognize the macro-organization of a passage and not get derailed by details or sub-topics.

 

Supporting Ideas and Details

Conversely, details questions do ask you to identify facts, details, descriptions or sub-topics. The details they ask about will always be explicitly stated, and sometimes the relevant part(s) of the passage will be highlighted to make it easier for you to find it. Thus, they’re often the easiest (or at least the most face-value) questions in a given set. They begin with phrases like “According to the passage” and “the passage states that.”

 

Inferences

Inference questions, on the other hand, ask you to identify what is implied by the author but not explicitly stated. Rather than “the passage states that,” these kinds of questions will begin with something like “the passage suggests that,” so you immediately know that you’ll need to connect the dots yourself. Nevertheless, the correct answer is always supported by the passage.

 

Out-of-Context

Out-of-context questions ask you to apply information from the passage to a different context. They often ask you to identify a parallel situation or analogous example. You’ll also see out-of-context questions asking whether the author would agree or disagree with something. Unlike inference questions, out-of-context questions use ideas or situations that aren’t from the passage.

 

Logical Structure and Organization

Logical structure questions center on function. Sometimes you’ll be asked to assess how a passage is constructed (does it define an idea, compare and contrast, refute an idea, etc.); other times you’ll be asked to recognize underlying assumptions, strengths, and weaknesses of the passage’s argument—as well as potential counterarguments.

Also falling under this category are questions that ask how the author persuades the reader—as opposed to what the author is persuading the reader of (which is more of a main idea question).

 

Style and Tone

Style and tone questions ask you about the author’s expression of ideas through diction, or word choice. You may have to deduce the author’s attitude toward a specific idea he/she brings up or identify the tone of the passage as a whole. Words like “critical” and “enthusiastic” often appear in the answer options for these kinds of questions.

 

These ladies have both style AND attitude—but not the kind we're talking about.
These ladies have both style AND attitude—but not the kind we’re talking about.

 

Should You Read the Passage or Question First?

Now that you’re familiar with the kinds of passages and questions that you’ll see, let’s talk about the order in which you should approach them.

The first option is to read the entire passage first, then attack each question. The second option is to read the first question first—just the question, not the answer choices—to get an initial idea of what you’re looking for before you start reading the passage. Feel free to practice both ways to see which one works best for you.

Ultimately, though, I recommend reading the passage first.

Why? Because you’re only able to see one question at a time on the GMAT. You might be able to skim the passage with that first question in mind and get to the right answer, but then the next question could be on a completely different facet of the passage, and you’ll be ill-prepared to answer it. Moreover, even a question that seems to be specific to one detail probably requires a holistic understanding of the passage in order to answer it correctly.

 

How to Approach the Passages: 7 GMAT Reading Comprehension Strategies

Nest up, let’s go over the best GMAT reading comprehension strategies for attacking the passages. After that, we’ll talk about the best GMAT reading comprehension tips for answering the questions, and then we’ll put all of this into action on a few of the real, official GMAT sample questions.

 

Practice Pacing Yourself

The Verbal section gives you 65 minutes for 36 questions, or a little under two minutes per question. But reading comprehension questions are going to take a bit longer than the critical reasoning questions, and significantly longer than the sentence correction questions. You should budget enough time to read these passages carefully—rather than skimming—in order to absorb all the important ideas and details.

That said, you don’t want to spend a ton of time decoding each word or sentence. If you really don’t “get” a sentence, don’t read over it more than twice, because you’ll lose a feel for the passage as a whole—you can come back to it if you need to for a specific question.

Plan your time accordingly: A good rule of thumb is to spend six minutes on a short passage with three questions, and eight minutes on a long passage with four questions. Two to three minutes should be spent on the initial reading, and the rest should be spent on the questions (including time to refer back to the passage as needed).

While you’re doing practice questions, consistently set a timer to get used to the recommended pace. This way, you can get timing down to an instinct before you take the real GMAT.

 

Pace yourself: the GMAT is a marathon, not a sprint!
Pace yourself: the GMAT is a marathon, not a sprint!

 

Watch for Transition Words and Phrases

Transition words track the relationship among ideas—as well as among the bits of reasoning and evidence that support or negate them. There are five main categories of transition words that you should know and recognize for the GMAT:

  • Cause-and-effect: words like “consequently,” “because,” and “thus” and phrases like “as a result.”
  • Agreeing/further expounding upon the same argument: words like “similarly” and “furthermore” and phrases like “in addition.”
  • Adding evidence: phrases like “for instance” and “in fact.”
  • Contrast or opposition: words like “yet,” “however,” and “nevertheless” and phrases like “on the other hand” or “in reality.”
  • Conclusion: Phrases like “all in all” and “in essence,” which indicate that the author is about to sum up or conclude their point. “Thus” can sometimes be used as a conclusion word as well.

Paying close attention to these words will help you follow the logical progression of a passage as you’re reading. Unless the passage actually centers on defining a concept, understanding the relationship among ideas and the author’s train of thought as the passage goes on is more important than doing a deep dive into what the ideas actually mean.

This is especially true if a passage is heavy in jargon (like natural science ones often are): the GMAT doesn’t care if you understand what “suprachiasmatic nuclei” is. They care that you can understand the role it plays in the passage’s main idea or thesis.

 

Abbreviate Jargon

Along these lines, you should abbreviate jargon and technical terms in your mind as you read. Any time you see a long technical term, a Latinate scientific name, or something similar, you should mentally shorten it to the first syllable. If it’s more than one word, abbreviate it into its initials. Either way, let your eyes gloss over the rest of the letters.

Many questions will abbreviate these terms anyway. For example, you might see something like “SN” in reference to “suprachiasmatic nuclei” in questions or answer options.

In general, you can think of jargon as a decoy: it’s there to distract you from tracking the logical progression of the passage and the point the author’s trying to make. Don’t fall into this trap!

 

Make a Mental Summary

GMAT reading comprehension passages often follow a typical structure: the main claim or point will be at the beginning, the next paragraph elaborates, and the next concludes. Even for ones that deviate slightly from this outline, you should still try to identify the main point as soon as possible—and then read each paragraph with an eye for how it builds or relates to the main point. After every paragraph, stop and take a moment to distill the “key idea” of the paragraph and fit it into your mental summary of what you’re reading. If the passage doesn’t have separate paragraphs, stop after every couple of sentences or right before a transition word.

Your mental summary should contain two parts: what and why.

  • What: What’s being said?
  • Why: Is the author trying to convince us of something or just laying out information? What’s the point of each paragraph, and why organize them that way?

One great trick is to simplify your summary as if you have to explain the passage to a little kid. This will help you distill complex passages down into their most straightforward, absorbable message.

 

Just like a chemist, you can "distill" complex reading passages down to their essence!
Just like a chemist, you can “distill” complex reading passages down to their essence!

 

Watch Out for Multiple Viewpoints

Sometimes a passage will contain multiple viewpoints—the viewpoint of the author and a viewpoint he/she disagrees with, multiple disparate viewpoints that the author is trying to mediate, etc. Take note of this: there will almost certainly be a question or two that tests your ability to distinguish, compare, or apply the different viewpoints presented. So these viewpoints should be a part of your mental summary: if they’re there, they’re there for a reason.

 

Find a Reason to Be Interested

It may seem counterintuitive, but to some extent, you really can choose whether or not you care about what you’re reading.

For example, let’s say you’re learning about bird migration patterns. Think about how cool it is that birds migrate in the first place. How amazing is it that they’re just born with the knowledge to fly thousands of miles south in the winter, and then back when it gets warm again up north? That’s so cool! Of course scientists want to understand how this works—and you do, too!

Okay, so I’m at risk of getting a little hokey here. But the point stands: find a way to engage with the passage. Find a way that it relates to the kind of knowledge that you might seek out in your own time. You’re more likely to absorb the passage well if you’re interested in what you’re reading.

 

Birds! They're neato.
Birds! They’re neato.

 

Take Notes

Most people think that the scratch board you’re given while taking the GMAT is just for the Quant section. This isn’t necessarily true: if there’s a single use case for it on the Verbal section, it’s reading comprehension. You don’t want to waste time writing down every detail, but on a passage-as-a-whole level, it can save time in the long run to jot down your mental summary as you read. You should then use your notes to help guide you on the questions.

Taking these kinds of notes is a good idea for most if not all students, but if you’re resistant to it, here are what I consider to be the three biggest use cases for taking notes:

  • You’re the kind of reader who finds it helpful to annotate the books you read for class.
  • In practice questions, you find that you’re consistently having a hard time answering “big picture” (main idea, inference, out-of-context, logical structure) questions.
  • Your eyeballs start to gloss over what you’re reading, and/or you find you have to read the passage multiple times to “get it.”

Of course, no one strategy works for every single student. Ultimately, you should practice both taking and not taking notes for the passages, and see which way works best for you.

 

Don’t Do This: Speed-Read or Skim

It bears repeating: GMAT reading comprehension questions require a thorough understanding of the passage. While you can (and should) simplify jargon in your mind as you read, you shouldn’t rush through the passage or just read the first and last sentence of every paragraph.

For one thing, you can’t see what you’re going to be asked beyond the very first question, which makes it hard to read predictively. You might unintentionally gloss over parts of the passage that will be important for a coming question. Moreover, as we just mentioned, the “next-level” questions (such as inference and out-of-context questions) will require a holistic, in-depth understanding of both the main idea and important details and how they interrelate. Overall, skimming and speed-reading can leave you poorly prepared for whatever lies ahead.

 

The Single Most Important Takeaway

Above all, the most important strategy for approaching the passages is to always read with the “main claim” or “main idea” in mind. Whether the author is making an argument or simply laying out information, there will always be a main idea that governs the passage’s content and organization.

 

body_viewpointsign
Watch out for the author’s viewpoint as you read!

 

4 GMAT Reading Comprehension Tips for Answering the Questions

Now that we’ve discussed how to approach reading the passages, let’s get into how to attack Reading Comprehension GMAT questions themselves.

 

#1: Make Sure You Answer the Question Being Asked

This may seem obvious, but the GMAT loves to give you answer options that are factually correct given the information in the passage but which don’t actually answer the question being asked. For example, if the question is asking you to identify an “assumption” behind a particular statement, the answer isn’t going to be something explicitly laid out in the passage. Likewise, if you’re being asked to identify a “main idea,” it’s not going to be a piece of supporting evidence.

 

#2: Stick to What You Can Prove With the Passage

Don’t bring in outside knowledge, even if you’re familiar with the topic being discussed. The “right” answer should always be 100% supportable by the passage alone.

 

#3: Read All the Answer Choices and Use Process of Elimination

Getting rid of wrong answers is an integral part of the process of getting to the right answer. Even if you’re absolutely 100% sure of your chosen answer, you should always take the time to briefly check the other options and make sure you can discount them before moving on.

This is also a good strategy for when you’re stuck between answer options: focus on disproving each one, and then go with the answer option that is hardest to disprove. It’s almost always easier to weed out the wrong answers than it is to find the right one.

 

#4: Familiarize Yourself with Incorrect Answer Types

In your test prep, don’t just find the right answer and move on: practice asking yourself why the other answer options are incorrect. You’ll start to see patterns in the wrong answers that are given, including:

  • Too broad: In main idea questions especially, you’ll need to watch out for answer options that take the main idea of the passage just a little bit too far. Sometimes this can hinge on a single word: something like “all,” “never,” or “every” that makes it too extreme of a statement given what’s in the passage.
  • Too narrow: A cousin of “too broad,” the “too narrow” answer options might get at a specific detail but not the main idea, making them incorrect answers for a main idea question.
  • Reversed causal or temporal relationship: Some answer options will “flip” a relationship. They’ll say that B caused A when the passage really states that A caused B, or they’ll say something came first in the process when it really came later on.
  • Not supported by the passage: The easiest to eliminate, these answer options bring in information or ideas that weren’t mentioned at all in the passage.
  • Supported by the passage but doesn’t answer the question being asked: As we said above, this is a very common wrong answer type, which is why it’s so important to make sure you fully understand what the question is asking before picking an answer. “Too narrow” falls into this broader category, but there are other kinds of decoy answer options that come up as well: options that are too much of an inference for a details question, options that are too explicit for an inference question, and so on.

Finally, let’s see these GMAT reading comprehension strategies in action with some sample questions.

 

body_doors
Eliminating all the wrong choices is how you get to the right one.

 

Reading Comprehension GMAT Examples

Check out out GMAT reading comprehension tips in action on the passage and questions below.

 

Passage

A meteor stream is composed of dust particles that have been ejected from a parent comet at a variety of velocities. These particles follow the same orbit as the parent comet, but due to their differing velocities they slowly gain on or fall behind the disintegrating comet until a shroud of dust surrounds the entire cometary orbit. Astronomers have hypothesized that a meteor stream should broaden with time as the dust particles’ individual orbits are perturbed by planetary gravitational fields. A recent computer-modeling experiment tested this hypothesis by tracking the influence of planetary gravitation over a projected 5,000-year period on the positions of a group of hypothetical dust particles. In the model, the particles were randomly distributed throughout a computer simulation of the orbit of an actual meteor stream, the Geminid. The researcher found, as expected, that the computer-model stream broadened with time. Conventional theories, however, predicted that the distribution of particles would be increasingly dense toward the center of a meteor stream. Surprisingly, the computer-model meteor stream gradually came to resemble a thick-walled, hollow pipe.

Whenever the Earth passes through a meteor stream, a meteor shower occurs. Moving at a little over 1,500,000 miles per day around its orbit, the Earth would take, on average, just over a day to cross the hollow, computer-model Geminid stream if the stream were 5,000 years old. Two brief periods of peak meteor activity during the shower would be observed, one as the Earth entered the thick-walled “pipe” and one as it exited. There is no reason why the Earth should always pass through the stream’s exact center, so the time interval between the two bursts of activity would vary from one year to the next.

Has the predicted twin-peaked activity been observed for the actual yearly Geminid meteor shower? The Geminid data between 1970 and 1979 shows just such a bifurcation, a secondary burst of meteor activity being clearly visible at an average of 19 hours (1,200,000 miles) after the first burst. The time intervals between the bursts suggest the actual Geminid stream is about 3,000 years old.

 

Passage Summary

Before I get into the questions, I’ll take a moment to summarize the passage:

  • Main idea: a computer simulation of the Geminid meteor stream has found that meteor streams broaden over time and form a hollow pipe shape.
    • Contrasts with conventional theories, which predicted broad, but centrally dense (not hollow)
    • Data from real Geminid meteor shower supports hollow shape theory: two bursts of showering when earth entered and exited the pipe

Note how I drilled the passage down to just the key ideas—this is more or less what your own notes and/or mental summary should look like for this passage.

 

Question 1

Which of the following is an assumption underlying the last sentence of the passage?

(A) In each of the years between 1970 and 1979, the Earth took exactly 19 hours to cross the Geminid meteor stream.

(B) The comet associated with the Geminid meteor stream has totally disintegrated.

(C) The Geminid meteor stream should continue to exist for at least 5,000 years.

(D) The Geminid meteor stream has not broadened as rapidly as the conventional theories would have predicted.

(E) The computer-model Geminid meteor stream provides an accurate representation of the development of the actual Geminid stream.

First, let’s look at the question and nail down exactly what it’s asking of us. This is a logical structure question, which we know because it’s asking us to identify an assumption underlying a statement in the passage. It has kindly directed us specifically to the last sentence: “The time intervals between the bursts suggest the actual Geminid stream is about 3,000 years old.”

Clearly, you can’t answer this question by reading the last sentence by itself—or even the last paragraph. You need to understand the context (what the author’s argument is) in order to correctly identify the implicit assumption at work here.

It’s hard to predict what exactly they’re looking for in this question, as there are many potential assumptions underlying that last sentence. So it doesn’t make sense in this particular question to try to come up with your own answer first. Let’s go straight to the answer choices instead:

(A) is a classic “supported by the passage but doesn’t answer the question” option. It restates something said explicitly in the passage, and it has to do with data that was discovered—not an underlying “assumption.” Eliminate it.

(B) brings in information that isn’t supported by the passage: we are never told about the state of the meteor, and even if we were, it wouldn’t seem to have any bearing on using time intervals between meteor shower bursts to measure the age of the stream. Eliminate it.

(C) takes a number that was brought up a few times—5,000 years—and misapplies it. 5,000 years was the projected period for the computer modeling experiment, not a prediction of how long a real comet stream “should” exist. Eliminate it.

(D) tests your understanding of how multiple theories compare and contrast in this passage. The conventional theory is that meteor streams will broaden and become denser at the center. The new experimental finding is that meteor streams will broaden and become a hollow, thick-walled pipe. Both of them agree on the broadening, just not on the specific shape. Thus, (D) intentionally misinterprets the experimental finding. Eliminate it.

(E) gives us some food for thought. The passage does imply—but never explicitly states—that the data from the real Geminid meteor showers supports the computer model’s findings. The computer model had predicted that if the stream were 5,000 years old, it would take earth 24 hours to pass through. It actually took earth 19 hours to pass through, which—according to the computer model—makes the stream about 3,000 years old.

“According to the computer model” is never stated; it’s exactly the assumption underlying the application of an age to the actual meteor stream. As in, the author has taken the meteor shower data as “proof” of the computer model’s validity, and then turned around and used the computer model to date the real thing. So (E) is correct.

 

body_meteorvector
It’s easy to stay engaged when you’re reading about meteors!

 

Question 2

According to the passage, why do the dust particles in a meteor stream eventually surround a comet’s original orbit?

(A) They are ejected by the comet at differing velocities.

(B) Their orbits are uncontrolled by planetary gravitational fields.

(C) They become part of the meteor stream at different times.

(D) Their velocity slows over time.

(E) Their ejection velocity is slower than that of the comet.

First, as always, let’s think about what the question is asking. It’s a details question, so we’ll have to find something explicitly stated in the passage.

Now, take a look at the answer options: they deal with words like “velocity,” “time,” and “gravity.” Where in the passage did they talk about these things? It was in the first paragraph. Let’s refer back to it, since the explanation of the physics of meteor dust was a bit tricky.

The passage states: “A meteor stream is composed of dust particles that have been ejected from a parent comet at a variety of velocities. These particles follow the same orbit as the parent comet, but due to their differing velocities they slowly gain on or fall behind the disintegrating comet until a shroud of dust surrounds the entire cometary orbit.”

Due to their differing velocities”—or, in simpler words, traveling at initial speeds that differ from each other—is the key here. Because they break off from the comet at different speeds, they gain ahead or fall behind it differently, ultimately landing in a variety of orbital positions and forming a kind of cloud. Choice (A) nails it, but we should still make sure we can eliminate the other options.

(B) directly opposes what’s stated in the passage: they are “perturbed” by planetary gravitational fields eventually.

(C) is a misinterpretation of the text—we’re dealing with velocity, not time.

(D) does get at the fact that the differing velocities don’t continue on forever, as the particles do “slowly gain or fall behind” the comet itself, but it doesn’t answer why the particles form a shroud around the comet, so it doesn’t answer the question being asked.

(E) brings in information never stated in the passage. All we know for certain is that the ejection velocities of the particles differ from each other, not the comet itself. Plus, some of the particles do “gain on” the comet, which implies that they’re traveling faster than the comet itself. So this answer option is at best speculative and at worst contrary.

We can safely stick with (A), the correct answer.

 

Did you know that shooting stars are actually bits and pieces of a meteor?
Did you know that shooting stars are actually bits and pieces of meteors?

 

Question 3

It can be inferred from the passage that which of the following would most probably be observed during the Earth’s passage through a meteor stream if the conventional theories mentioned in line 18 were correct?

(A) Meteor activity would gradually increase to a single, intense peak, and then gradually decline.

(B) Meteor activity would be steady throughout the period of the meteor shower.

(C) Meteor activity would rise to a peak at the beginning and at the end of the meteor shower.

(D) Random bursts of very high meteor activity would be interspersed with periods of very little
activity.

(E) In years in which the Earth passed through only the outer areas of a meteor stream, meteor activity would be absent.

This is clearly an inference question. The author is asking us to hypothesize about what would happen if the conventional theories were true as opposed to the experimentally proven one. It’s not an out-of-context question, as it’s asking us about a situation within the passage, not outside of it.

So, given that the passage gives us the answer for the new theory, can we extrapolate from that what would happen for the conventional theory? First, we have to identify what the conventional theory really is. Here’s where that passage summary really comes in handy: We have distilled that the computer model (and ultimately supported) theory is the ‘hollow pipe’ theory, and that the conventional theory is the ‘centrally dense’ theory. If bursts happen when earth enters and exits the walls of the pipe, what would happen if earth passed through a centrally dense formation?

(A) makes perfect sense. The showers would gradually increase to a peak at the center, and then decrease again. All of the other answer options point to a different shape: (B) implies that the stream is uniform throughout; (C) reverses the pattern, matching more closely to the hollow “walls” theory than to the conventional theory; (D) implies a random shape; and (E) implies that there are no particles at all in the outer regions, rather than just a less-dense clustering of them.

 

This T-Rex is a meat-eater, not to be confused with meteor (I'll be here all night).
This T-Rex is a meat-eater, not to be confused with meteor (I’ll be here all night).

 

GMAT Reading Comprehension: Key Takeaways

TL; DR? You’ll encounter roughly 9 to 16 reading comprehension questions on the GMAT. While the questions vary, they all essentially test your ability to understand an argument or identify the important ideas and information from a given passage. You should:

  • Read the passage carefully.
  • Look out for the main idea/main viewpoint, and for the interrelationship among sub-concepts/other viewpoints.
  • Pay special attention to transition words, but gloss over jargon.
  • Make a written or mental summary as you read.
  • Attack the questions by using your summary and eliminating wrong answers strategically.

No matter what, the most important thing is to nail down the best mix of the above strategies and tactics for you well before you take the real GMAT. Don’t deviate from your plan on test day: sticking to the tried-and-true approaches will ensure that you do your best.

 

What’s Next?

Now that you’re an expert on reading comprehension GMAT questions, check out these similar posts on the best strategies for sentence correction questions and critical reasoning questions. (coming soon)

Wondering about the best way to study for the GMAT? Check out these tips for designing your GMAT study plan. (coming soon)

Other than reading comprehension, are you familiar with the format and question types on the rest of the GMAT? Our guide to the full GMAT exam pattern has samples of each and every question type.

When you’re ready to start taking practice tests, read through our guide to the best 28 GMAT practice tests to help you prepare for the exam. Good luck!

The post The 11 Best GMAT Reading Comprehension Strategies appeared first on Online GMAT Prep Blog by PrepScholar.

]]>
The 5 Hardest GMAT Data Sufficiency Questions /gmat/blog/hardest-gmat-data-sufficiency-questions/ Fri, 22 Sep 2017 20:00:31 +0000 http://gmat.psblogs.com/?p=3658 For many GMAT test takers, Data Sufficiency questions are the most difficult questions on the GMAT. But what do the hardest GMAT Data Sufficiency questions look like? What skills and concepts do they test? What do they have in common? What Data Sufficiency strategies can we use to get these challenging GMAT Quantitative questions right? … Continue reading "The 5 Hardest GMAT Data Sufficiency Questions"

The post The 5 Hardest GMAT Data Sufficiency Questions appeared first on Online GMAT Prep Blog by PrepScholar.

]]>
pexels-photo-249360

For many GMAT test takers, Data Sufficiency questions are the most difficult questions on the GMAT. But what do the hardest GMAT Data Sufficiency questions look like? What skills and concepts do they test? What do they have in common? What Data Sufficiency strategies can we use to get these challenging GMAT Quantitative questions right?

In this article, I’ll go over the five hardest GMAT Data Sufficiency questions, what you’ll need to know to solve them, how to approach them on test day, and what we can learn from hard GMAT Quant questions about mastering Data Sufficiency.

 

How We Found These GMAT Data Sufficiency Questions

To gather the hardest GMAT questions, our GMAT experts took advantage of the computer adaptive algorithm used on the test. Over the course of the test, the difficulty levels of questions change based on how well you performed on previous questions. Get a few questions right, move up a difficulty level. Get a few questions wrong, move down a difficulty level. By the end of the test, every test taker should be presented with questions that perfectly match their ability.

Our GMAT experts took the practice tests on the GMATPrep software multiple times without missing a single question on the Quantitative section. We collected the questions they received into a master list of the hardest GMAT Quantitative questions. We then looked at activity on various online forums to determine which of these hard GMAT math questions test takers struggled with the most from each question type. This left us with the five hardest GMAT Data Sufficiency questions out there, ready for you to study!

 

math-1500720_640

 

GMAT Data Sufficiency Question 1

The points $A$,$B$,$C$, and $D$ are on a number line, not necessarily in that order. If the distance between $A$ and $B$ is 18 and the distance between $C$ and $D$ is 8, what is the distance between $B$ and $D$?

  1. The distance between $C$ and $A$ is the same as the distance between $C$ and $B$.
  2. $A$ is to the left of $D$ on the number line.
  1. Statement (1) ALONE is sufficient, but statement (2) alone is not sufficient.
  2. Statement (2) ALONE is sufficient, but statement (1) alone is not sufficient.
  3. BOTH statements TOGETHER are sufficient, but NEITHER statement ALONE is sufficient.
  4. EACH statement ALONE is sufficient.
  5. Statements (1) and (2) TOGETHER are NOT sufficient.

This particular problem gives us four different numbers on a number line ($A$, $B$, $C$, and $D$) and tells us the distance between two sets of points ($A$ ↔ $B$ and $C$ ↔ $D$). We should also note that these points are not necessarily in alphabetical order. Whenever we have GMAT Quant questions dealing with shapes, graphs, number lines, etc., it’s a really good call to draw out examples — this is the visual equivalent of plugging in numbers.) Applying this trick here, and remembering that the distance between $A$ and $B$ is longer than between $C$ and $D$, we see that our line could look like this:
Screen Shot 2017-09-19 at 11.32.35 AMthis:
Screen Shot 2017-09-19 at 11.30.04 AMthis:
Screen Shot 2017-09-19 at 11.34.17 AMand so on.

We need to find the distance between $B$ and $D$. This means that we need to gather information

  1. about the order of the points
  2. about how our first set of points ($A$ and $B$) relate to our second set of points ($C$ and $D$)

 

Statement 1

If $A$ and $B$ are two different points and are both the same distance from $C$, this means that the distance between $A$ and $C$ must also be 18 and that $C$ must be directly between the two points like so:
Screen Shot 2017-09-19 at 11.43.33 AMWe also know that $D$ is only 8 away from $C$, so it is closer to $C$ than either $A$ or $B$. However, we still don’t know where $D$ is compared to these points. It could be between points $A$ and $C$, making it 26 away from $B$:
Screen Shot 2017-09-19 at 11.43.02 AMor between points $C$ and $B$, making it 10 away from $B$:
Screen Shot 2017-09-19 at 11.42.23 AMSince we don’t know whether the distance between $B$ and $D$ is 26 or 10, Statement 1 is insufficient.

 

Statement 2

Statement 2 tells us that $A$ is to the left of $D$. Well, $A$ is to the left of $D$ in both of the number lines above, and the distance between $B$ and $D$ is not the same in either. So this doesn’t tell us much. If the statement told us that $A$ was directly to the left of $D$, this might be a little more helpful … but it didn’t and it isn’t. Statement 2 is insufficient.

 

BOTH

Well, we already established that $A$ is to the left of $D$ (fulfilling Statement 2) in both of the number lines we created to fulfilling Statement 1, so even with the information from both statements, we don’t know whether the distance between $B$ and $D$ is 26 or 10. Since we still can’t solve for a single solution, the correct answer is E: Statements 1 and 2 TOGETHER are NOT sufficient to answer the question.

 

measurement-1476919_640

 

GMAT Data Sufficiency Question 2

If Bob produces 36 or fewer items in a week, he is paid $x$ dollars per item. If Bob produces more than 36 items in a week, he is paid $x$ dollars per item for the first 36 items and $1{1/2}$ times that amount for each additional item. How many items did Bob produce last week?

  1. Last week Bob was paid a total of \$480 for the items that he produced that week.
  2. This week Bob produced 2 items more than last week and was paid a total of \$510 for the items that he produced this week.
  1. Statement (1) ALONE is sufficient, but statement (2) alone is not sufficient.
  2. Statement (2) ALONE is sufficient, but statement (1) alone is not sufficient.
  3. BOTH statements TOGETHER are sufficient, but NEITHER statement ALONE is sufficient.
  4. EACH statement ALONE is sufficient.
  5. Statements (1) and (2) TOGETHER are NOT sufficient.

Whenever we have a word problem, like this one, we want to translate the words into math. Scanning over the problem, we see the phrases “36 or fewer” and “more than 36” — these are classic signs that we’re dealing with inequalities. This particular problem gives us two scenarios for calculating how much Bob is paid based on how many total items he produces in a given week (one for 36 or fewer items, one for more than 36 items), so we want to create two equations: one for each scenario. Letting $i$ = the number of items Bob makes in a given week, we can translate our first scenario as

$\If i ≤ 36\, \then \total \pay=x×i$

Our second sentence is a little more complicated. If Bob produces more than Bob is paid $x$ for the first 36 items (or $36x$). Then for all of the items after 36 (or $i-36$), he is paid $1.5x$ (or $1.5x×(i-36)$). Putting that together,

$\If i > 36\, \then \total \pay=x×36 + 1.5x×(i-36)$

So we have two equations, each with three variables ($i$, $x$, and $\total \pay$) … which means we need a bunch of information to figure out an answer. To figure out a value for $i$, we need information about

  • which of the two equations to use
  • the value of $x$
  • the total pay

 

Statement 1

This statement tells us how much Bob was paid last week, but it doesn’t tell us anything about the specific value of $x$ or which of the two equations we should use. So we could have:

$i=1 \and x=480 → 480=480×1$

or

$i=32 \and x=15 → 480=15×32$

or

$i=76\ \and x=5 → 480=5×36 + 1.5(5)×(40)$

and so on. Statement 1 is insufficient.

 

Statement 2

This one tells us how much Bob was paid this week, and it compares the number of items he produced this week to the number he produced last week. Well, we don’t know anything about how many items Bob produced last week, so the last piece of information doesn’t tell us much about $x$ — he could have produced 1 item last week and 3 this week or 100 items last week and 102 this week. And, like in Statement 1, we don’t know whether or not $i$ is greater than 36, so we don’t know which statement to use. So we could have:

$i=4 \and x=145 → 580=145×4$

or

$i=29 \and x=20 → 580=20×29$

or

$i=41\, \and x=13{1/3} → 580=13{1/3}×36 + 1.5(13{1/3})×(5)$

and so on. Statement 2 is insufficient.

 

BOTH

What if we put the two statements together? Well, now we know something: the additional two items Bob produced this week earned him \$30 more than he earned last week. This means that Bob earned an extra /$15 per item. But we’re still missing a key piece of information: which scenario are we dealing with?

  1. Did Bob produce 36 or fewer items this week? If so, then both items were produced at a rate of $x$, so that $x=15$.
  2. Did Bob produce at least 38 items this week? If so, then both items were produced at a rate of $1.5x$, so that $1.5x=15$ → $x=10$?
  3. OR did Bob produce exactly 35 items last week and 37 items this week? If so, then the first item was produced at a rate of $x$ and the second item was produced at a rate of $1.5x$, so that $x+1.5x=30$ → $2.5x=30$ → $x=12$.

We’ve got a few options here, so let’s try each individually. Remember, we want to solve for the number of items Bob produced last week, so we’ll use that equation:

  1. $x=15$, $480=15i$ → $i=32$
  2. $x=10$, $480=36(10)+1.5(10)(36-i)$ → $480=360+15(36-i)$ → $120=15(i-36)$ → $8=i-36$ → $i=44$

We already have two possible solutions, so we don’t need to look at our third, more complicated option. We cannot determine whether Bob made 32 or 44 items last week, so we cannot solve the problem with both statements. The correct answer is E: Statements 1 and 2 TOGETHER are NOT sufficient to answer the question.

 

craftsman-19791_640

 

GMAT Data Sufficiency Question 3

Of the 75 houses in a certain community, 48 have a patio. How many of the houses in the community have a swimming pool?

  1. 38 of the houses in the community have a patio but do not have a swimming pool.
  2. The number of houses in the community that have a patio and a swimming pool is equal to the number of houses in the community that have neither a swimming pool nor a patio.
  1. Statement (1) ALONE is sufficient, but statement (2) alone is not sufficient.
  2. Statement (2) ALONE is sufficient, but statement (1) alone is not sufficient.
  3. BOTH statements TOGETHER are sufficient, but NEITHER statement ALONE is sufficient.
  4. EACH statement ALONE is sufficient.
  5. Statements (1) and (2) TOGETHER are NOT sufficient.

Reading through the question, we see that we’re dealing with a group of houses where some have a swimming pool and some have a patio. Scanning over the statements, we see that some houses have only a pool, some houses have only a patio, some have neither, and some have both. Almost anytime we see the word “both” in GMAT Quant questions, we’re dealing with an overlapping sets problem — we are looking at two criteria (here, having a pool and having a patio) and where they overlap (here, having “both” a pool and a patio).

Overlapping sets problems have a lot of information, so it’s really easy to get lost in them. A good trick is to use a visual representation to keep track of what you know:

  • For two overlapping criteria, use a table, where each axis represents one criterion.
  • For three overlapping criteria, use a venn diagram, where each circle represents a criterion.

Here, we have two overlapping sets, so we’re going to use a table. We’ll go ahead and fill in only what was stated directly in the question. We want to find the total number of houses that have a Pool, so we’ll represent that in our table as $x$:

Patio No Patio Total
Pool $x$
No Pool  
Total 48    75

Because of the way we’ve set the table up, the two numbers in each row should add up to the total at the end of the row and the two numbers in each columns should add up to the total at the bottom of the column. This means that if we have at least two of the three values in each row or column, we should be able to solve for the third. Looking at our table, we see that our total row along the bottom has two values. If there are 75 houses in total and 48 of those houses have patios, 75 – 48 = 27 of those houses must not have patios. We can go ahead and fill that information in our table:

Patio No Patio Total
Pool $x$
No Pool  
Total 48 27   75

Doesn’t seem like we can get much more out of our table at this point, so we’ll move on to our Statements.

 

Statement 1

To start we’ll fill in the information directly given in the statement:

Patio No Patio Total
Pool $x$
No Pool 38  
Total 48 27   75

We see that our first column has two values, so we should be able to solve for the third. If there are 58 houses with patios and 38 of those houses do not have pools, 48 – 38 = 10 of those houses must have pools:

Patio No Patio Total
Pool  10 $x$
No Pool  38  
Total 48 27   75

Looking at the row and the column that contain $x$, we see that we only have one number value for each, meaning that we can’t solve for $x$. Statement 1 is insufficient.

 

Statement 2

This statement doesn’t give us any concrete numbers to work with, but it does tell us that two of our values (houses with both pools and patios and houses with neither pools nor patios) are equal to each other. When we know that the same number shows up in two places, but we don’t know what that number is, it’s a good idea to represent that number with a variable — if we represent both values as, say, $n$, we know that they are the same number and can combine or eliminate them down the line:

Patio No Patio Total
Pool  $n$ $x$
No Pool  $n$  
Total 48 27   75

Now we’re getting somewhere! We don’t have two number values in any row or column, but we can use both the top row and the second column to represent No Patio/Pool with variables: if there are $x$ total houses with pools and $n$ of those houses have patios, $x-n$ must not have patios, and if there are 27 total houses that do not have patios, and $n$ of those houses do not have pools, $27-n$ must have pools:

Patio No Patio Total
Pool $n$ $x-n$ or $27-n$ $x$
No Pool $n$  
Total 48 27   75

Since the number of houses with no patio and a pool equals both $x-n$ and $27-n$, we can set the two equal to each other to solve for $x$:

$x-n=27-n$

$x=27$

We were able to determine that 27 houses have pools, which means that Statement 2 is sufficient. The correct answer is B: Statement 2 alone is sufficient to answer the question.

 

pool-1567760_640

 

GMAT Data Sufficiency Question 4

Each employee of Company Z is an employee of either Division X or Division Y, but not both. If each division has some part-time employees, is the ratio of the number of full-time employees to the number of part-time employees greater for Division X than for Company Z?

  1. The ratio of the number of full-time employees to the number of part-time employees is less for Division Y than for Company Z.
  2. More than half of the full-time employees of Company Z are employees of Division X, and more than half of the the part-time employees of Company Z are employees of Division Y.
  1. Statement (1) ALONE is sufficient, but statement (2) alone is not sufficient.
  2. Statement (2) ALONE is sufficient, but statement (1) alone is not sufficient.
  3. BOTH statements TOGETHER are sufficient, but NEITHER statement ALONE is sufficient.
  4. EACH statement ALONE is sufficient.
  5. Statements (1) and (2) TOGETHER are NOT sufficient.

Right away, the word “ratio” tips us off that we’re dealing with ratios in this problem, and the word “greater” indicates that we’re dealing with inequalities. However, as we read through the rest of the problem, things start to get a little more confusing: one company, two divisions, full-time and part-time employees … this is a lot to process.

We do see the words “either” and “both” though, which should get some overlapping sets wheels turning in our minds. We see that, like the problem above, we have two criteria: employees can belong to Division X or Division Y and can be full-time or part-time. Since this problem doesn’t have any concrete numbers, it isn’t strictly necessary to make a table like we did in the problem above. However, it can still be helpful to define the relationships between our sets and build equations:

Division X Division Y Total (Company Z)
Full-Time
Part-Time  
Total (employees)  

We know that the two numbers in each row should add up to the total at the end of the row and the two numbers in each columns should add up to the total at the bottom of the column. So we can now build 6 different equations:

  1. $\Full\-\Time \@ \X + \Part\-\Time \@ \X = \Employees \@ \X$
  2. $\Full\-\Time \@ \Y + \Part\-\Time \@ \Y = \Employees \@ \Y$
  3. $\Full\-\Time \@ \Z + \Part\-\Time \@ \Z = \Employees \@ \Z$
  4. $\Full\-\Time \@ \X + \Full\-\Time \@ \Y = \Full\-\Time \@ \Z$
  5. $\Part\-\Time \@ \X + \Part\-\Time \@ \Y = \Part\-\Time \@ \Z$
  6. $\Employees \@ \X + \Employees \@ \Y = \Employees \@ \Z$

Now that we have this set up, let’s figure out what the question is asking for. Like with all word problems, we want to translate words into math. Whenever we’re dealing with ratios, we should remember that ratios can (and should) be expressed as fractions:

Is ${\full\-\time \@ \X}/{\part\-\time \@ \X} > {\full\-\time \@ \Z}/{\part\-\time \@ \Z}$?

or in other words, are there more full-time employees for every part-time employee at Division X than at the entire company?

 

Statement 1

This Statement gives us information about the ratio of full-time employees to part-time employees at Division Y compared to Company Z:

${\full\-\time \@ \Y}/{\part\-\time \@ \Y} < {\full\-\time \@ \Z}/{\part\-\time \@ \Z}$

Now, before we rule this statement out because it doesn’t tell us anything about Company X, let’s see how we can use our equations to substitute X back into the inequality. Looking at equations 4 and 5, we see that we can rearrange the equations to give:

  1. $\Full\-\Time \@ \Y = \Full\-\Time \@ \Z – \Full\-\Time \@ \X$
  2. $\Part\-\Time \@ \Y = \Part\-\Time \@ \Z – \Part\-\Time \@ \X$

Subbing those into our inequality gives us:

${\full\-\time \@ \Z – \full\-\time \@ \X}/{\part\-\time \@ \Z – \part\-\time \@ \X} < {\full\-\time \@ \Z}/{\part\-\time \@ \Z}$

Let’s think about what we know about fractions. To make a fraction smaller, we need to either

  1. decrease the numerator relative to the denominator
  2. increase the denominator relative to the numerator

We know that we are decreasing both the numerator and denominator, so we must be decreasing the numerator by a greater percentage than we are decreasing the denominator. This means that the number of full-time employees at Division X is larger relative to the number of part-time employees at Division X than the number of full-time employees at Company Z to the number of part-time employees at Company Z. In other words, the ratio of the number of full-time employees to the number of part-time employees is greater for Division X than for Company Z. Statement 1 is sufficient.

 

Statement 2

Like with Statement 1, let’s translate this into math:

$\full\-\time \@ \X > {1/2}\full\-\time \@ \Z$

$\part\-\time \@ \Y > {1/2}\part\-\time \@ \Z$

Given equation 5, the second half of our statement also tells us that

$\part\-\time \@ \X < {1/2}\part\-\time \@ \Z$

This means we can write the ratio of full-time employees at Division X as

${>{1/2}\full\-\time \@ \Z}/{<{1/2}\part\-\time \@ \Z}$

or, cancelling the {1/2} in both the numerator and denominator,

${>\full\-\time \@ \Z}/{<\part\-\time \@ \Z}$

To make a fraction larger, we need to either:

  1. increase the numerator relative to the denominator
  2. decrease the denominator relative to the numerator

Here, we’re doing both: full-time employees at Division X is greater than full-time employees at Company Z and part-time employees at Division X is less than part-time employees at Company Z. This means that

${\full\-\time \@ \X}/{\part\-\time \@ \X} > {\full\-\time \@ \Z}/{\part\-\time \@ \Z}$

which is exactly what we’re trying to solve for. Statement 2 is sufficient.

Since both statements are sufficient to solve the problem individually, the correct answer is D.

 

men-1979261_640

 

GMAT Data Sufficiency Question 5

If $n$ is a positive integer and $r$ is the remainder when $(n-1)(n+1)$ is divided by 24, what is the value of $r$?

  1. $n$ is not divisible by 2
  2. $n$ is not divisible by 3
  1. Statement (1) ALONE is sufficient, but statement (2) alone is not sufficient.
  2. Statement (2) ALONE is sufficient, but statement (1) alone is not sufficient.
  3. BOTH statements TOGETHER are sufficient, but NEITHER statement ALONE is sufficient.
  4. EACH statement ALONE is sufficient.
  5. Statements (1) and (2) TOGETHER are NOT sufficient.

The word “remainder” tells us that we’re dealing with, what else, a remainder problem. Remainder problems scare a lot of students because they don’t involve an easy to use/memorize formula. However, this means that we have a great opportunity to plug in numbers.

Even though this isn’t technically a “word problem”, we still need to translate the words into math to build an equation:

${(n-1)(n+1)}/24 = \? \| \R\: r$

Let’s make a note that $n$ must be a positive integer and move on to our statements.

 

Statement I

This statement tells us that $n$ is not divisible by two — in other words, it’s telling us that $n$ is odd. Let’s try plugging in numbers. When we select numbers to plug in, our goal is to prove that the statement is insufficient: in other words, we want to pick numbers that will give us different results. We also want to pick numbers that are easy to work with to save time.

We see that one of the values in our numerator is (n-1), which means that picking 1 will give us a zero in our numerator. That seems like it’ll give us an interesting result, so we’ll give it a shot:

${(1-1)(1+1)}/24$

${(0)(2)}/24$

$0/24$

$0 | \R\: 0$

So when $n=1$, $r=0$. Let’s try our next odd number up, $3$ — based on the size of the denominator, it seems like our numerator will be smaller than the denominator, giving a solution of 0 with positive remainder:

${(3-1)(3+1)}/24$

${(2)(4)}/24$

$8/24$

$0 | \R\: 8$

So when $n=3$, $r=8$. This means that $r$ can be either 0 or 8 given Statement 1. Since we can’t find a single value for $r$, Statement 1 is insufficient.

 

Statement II

This statement tells us that $n$ is not divisible by three. That knocks $n=3$ out of the running. $n=1$ still works, however, so we know that $r=0$ is still a possibility given Statement 2.

Since we tried only odd numbers last time, let’s try an even number this time to see if that changes things up: we’ll do 2 to keep our numbers easy to work with:

${(2-1)(2+1)}/24$

${(1)(3)}/24$

$3/24$

$0 | \R\: 3$

So when $n=2$, $r=3$. This means that $r$ can be either 0 or 3 given Statement 2. Like before, since we can’t find a single value for $r$, Statement 2 is insufficient.

 

BOTH

Putting these two statements together, we know that $n$ must be odd and cannot be divisible by 3: so we have 1, 5, 7, 11, etc. These numbers are going to get pretty big pretty fast, so let’s try them from smallest to greatest. We already know that $r=0$ when $n=1$, so we want to find a positive value for $r$ to prove that both statements are insufficient:

${(5-1)(5+1)}/24$

${(4)(6)}/24$

$24/24$

$1 | \R\: 0$

So when $n=5$, $r=0$. That’s the same as when $n=1$. Let’s try the next number up, 7:

${(7-1)(7+1)}/24$

${(6)(8)}/24$

$48/24$

$2 | \R\: 0$

So when $n=7$, $r=0$. We’re starting to see the hints of a pattern here. Let’s try one more, 11, to be sure:

${(11-1)(11+1)}/24$

${(10)(12)}/24$

$120/24$

$5 | \R\: 0$

So when $n=7$, $r=0$. Once we’ve tried at least 4 numbers in a series and confirmed that we’ve done a reasonable job picking numbers that would give us different results, we can usually determine that we have a pattern. Here, we can say confidently that given Statement 1 and Statement 2, $r$ will always be 0. This means that the correct answer is C: BOTH statements together are sufficient.

 

learn-2300141_640

 

Key Takeaways: Learning From The Hardest Data Sufficiency Questions

So what can the hardest GMAT Quantitative questions teach us about GMAT Data Sufficiency questions in general?

  1. Visuals — drawings, tables, Venn diagrams, graphs, what have you — are our friends, and not only on Geometry questions. On the GMAT, advanced quant questions are hard to conceptualize, and drawing things out keeps us from having to keep track of a lot of complicated relationships in our heads.
  2. Whenever we have words, we need to translate them into math. Like visuals, building equations helps us take hard GMAT math questions and distill them into something we can work with. Use math-y keywords, like “greater than”, “equal to”, “divided by”, etc. to break sentences down into their component parts.
  3. The hardest GMAT Data Sufficiency questions often involve more logic than simple math, especially around number sense concepts. Being comfortable making inferences based on what we know can save us a lot of time compared to slogging through a bunch of proofs.
  4. That said, picking numbers to plug in is a great Data Sufficiency strategy that can help us avoid overthinking a problem or confirm our logic. Always pick numbers that you think will yield two different solutions, making the statement insufficient.

 

What’s Next?

What are the math concepts tested on the GMAT? The best GMAT math tricks and shortcuts? The most important Data Sufficiency tips? These articles expand on the concepts used in these five problems, explaining what you need to know about GMAT Data Sufficiency before test day.

Looking to improve your Quant score? This article explains what exactly a good GMAT Quantitative score is.

If you’d like similar analyses of the hardest questions from other GMAT question types, check out our post on the five hardest Sentence Correction questions.

The post The 5 Hardest GMAT Data Sufficiency Questions appeared first on Online GMAT Prep Blog by PrepScholar.

]]>
The 5 Hardest GMAT Sentence Correction Questions /gmat/blog/hardest-gmat-sentence-correction-questions/ Thu, 21 Sep 2017 05:00:02 +0000 http://gmat.psblogs.com/?p=3643 For many GMAT test takers, hard Sentence Correction questions are what stand between them and a top Verbal score. But what do the hardest GMAT Sentence Correction questions look like? What skills and concepts do they test? What do they have in common? What strategies can we use to get these GMAT Verbal questions right? … Continue reading "The 5 Hardest GMAT Sentence Correction Questions"

The post The 5 Hardest GMAT Sentence Correction Questions appeared first on Online GMAT Prep Blog by PrepScholar.

]]>
correcting-1870721_640

For many GMAT test takers, hard Sentence Correction questions are what stand between them and a top Verbal score. But what do the hardest GMAT Sentence Correction questions look like? What skills and concepts do they test? What do they have in common? What strategies can we use to get these GMAT Verbal questions right?

In this article, I’ll go over the five hardest GMAT Sentence Correction questions, what you’ll need to know to solve them, how to approach them on test day, and what we can learn from hard GMAT questions about mastering Sentence Correction.

 

How We Found These GMAT Sentence Correction Questions

To gather these questions, our GMAT experts took advantage of the computer adaptive algorithm used on the test. Over the course of the test, the difficulty levels of questions change based on how well you performed on previous questions. Get a few questions right, move up a difficulty level. Get a few questions wrong, move down a difficulty level. By the end of the test, every test taker should be presented with questions that perfectly match their ability.

Our GMAT experts took the practice tests on the GMATPrep software multiple times without missing a single question on the Verbal section. We collected the questions they received into a master list of the hardest GMAT Verbal questions. We then looked at activity on various online forums to determine which of these questions test takers struggled with the most from each question type. This left us with the five hardest GMAT Sentence Correction questions out there, ready for you to study!

 

pen-writing-notes-studying

 

GMAT Sentence Correction Question 1

Based on records from ancient Athens, each year young Athenian women collaborated to weave a new woolen robe that they used to dress a statue of the goddess Athena and that this robe depicted scenes of a battle between Zeus, Athena’s father, and giants.

  1. Based on records from ancient Athens, each year young Athenian women collaborated to weave a new woolen robe that they used to dress
  2. Based on records from ancient Athens, each year young Athenian women had collaborated to weave a new woolen robe with which to dress
  3. According to records from ancient Athens, each year young Athenian women collaborated to weave a new woolen robe that they used to dress
  4. Records from ancient Athens indicate that each year young Athenian women collaborated to weave a new woolen robe with which they dressed
  5. Records from ancient Athens indicate each year young Athenian women had collaborated to weave a new woolen robe for dressing

When we have a sentence as long and confusingly ordered as this one, a really good move is to eliminate anything unnecessary to the grammatical structure of the sentence. This can help us to figure out what exactly the sentence is actually saying and pinpoint errors in sentence construction. Some good candidates for elimination are parts of the sentence set off by commas and long descriptive phrases — we should be able to remove anything that provides “extra” information and still have a grammatical sentence. For example,

This Thursday afternoon, I went to the potluck at Julianne and Michelle’s apartment.

can be simplified to

This Thursday afternoon, I went to the potluck at Julianne and Michelle’s apartment.

without messing up the grammar.

Let’s do the same thing with our sentence:

Based on records from ancient Athens, each year young Athenian women collaborated to weave a new woolen robe that they used to dress a statue of the goddess Athena and that this robe depicted scenes of a battle between Zeus, Athena’s father, and giants.

or, simplified,

Young Athenian women collaborated to weave a new woolen robe and that this robe depicted scenes of a battle.

Much simpler! Now we see that the sentence is divided into a series of two parts (“Young … woolen robe” and “that … battle”) separated by a conjunction (“and”). We also see that the last portion of the sentence is not underlined. Whenever we see that we have a series where part of the series is underlined and part is not, we should immediately start looking for parallelism issues.

Looking at our two parts, we see one big difference in grammatical structure:

  • Young Athenian women collaborated to weave a new woolen robe
  • that this robe depicted scenes of a battle

Since the part of our series that isn’t underlined begins with “that”, we need the other part to include “that” to match. Looking through the answer options, only D uses the word “that” twice, giving us correct parallel structure.

 

monument-2011140_640

 

GMAT Sentence Correction Question 2

With surface temperatures estimated at minus 130 degrees Fahrenheit, Jupiter’s moon Europa has long been considered far too cold to support life, and with 60 square miles of water thought to be frozen from top to bottom.

  1. Europa has long been considered far too cold to support life, and with
  2. Europa has long been considered far too cold to support life, its
  3. Europa has long been considered as far too cold to support life and has
  4. Europa, long considered as far too cold to support life, and its
  5. Europa, long considered to be far too cold to support life, and to have

This question is tricky because it relies heavily on having an idiom memorized rather than applying grammatical concepts — if you don’t know the idiom, it’s going to be much harder to get the question right. The GMAT has a few classic idioms that are bound to show up on any test. “Consider X, Y” is one of them. “Consider as” and “consider to be” are both used colloquially, but neither is grammatically correct. This allows us to eliminate C, D, and E right off the bat, leaving us with A and B.

Like the previous question, we’re dealing with a pretty complex sentence. Right away, we see that “With surface … Fahrenheit” is a descriptive phrase set off by a comma, so it shouldn’t be necessary to the grammar of the sentence. Let’s go ahead and eliminate it from our sentence to simplify things:

Jupiter’s moon Europa has long been considered far too cold to support life, and with 60 square miles of water thought to be frozen from top to bottom.

Looking at A, we see that the sentence is divided into two clauses (“Jupiter’s … life” and “with … bottom”) separated by a conjunction (“and”). However, this time we have a comma before our conjunction. Whenever we have two clauses connected by a comma + a conjunction, both clauses need to be independent — they need to be able to stand on their own as sentences. For example,

Rosalind put on boots and wrapped a scarf around her neck.

is fine, but

Rosalind put on boots, and wrapped a scarf around her neck.

is not — “wrapped a scarf around her neck” doesn’t work as it’s own sentence. We would need to add a subject:

Rosalind put on boots, and she wrapped a scarf around her neck.

Let’s see if both parts of our sentence can stand alone as independent clauses:

  • Europa has long been considered far too cold to support life.
  • With 60 square miles of water thought to be frozen from top to bottom.

We see right away that the second part of our sentence is not a complete thought. We can eliminate A, leaving us with B as the correct answer.

Now, if we didn’t recognize the idiom error in the sentence, we can eliminate A based on the reasoning above and continue to use sentence structure to eliminate answers. We see that a few of our answer choices have additional pieces of the sentence set off by commas that we can remove:

  1. Jupiter’s moon Europa has long been considered far too cold to support life, its 60 square miles of water thought to be frozen from top to bottom.
  1. Jupiter’s moon Europa, long considered as far too cold to support life, and its 60 square miles of water thought to be frozen from top to bottom.
  2. Jupiter’s moon Europa, long considered to be far too cold to support life, and to have 60 square miles of water thought to be frozen from top to bottom.

As expected, B looks fine. However, we see that D and E don’t make much sense once we remove the “unnecessary” parts of the sentence. We can eliminate both, leaving us with B, C, and a 50% chance of guessing correctly.

 

astronomy-2064273_640

 

GMAT Sentence Correction Question 3

For the farmer who takes care to keep them cool, providing them with high-energy feed, and milking them regularly, Holstein cows are producing an average of 2,275 gallons of milk each per year.

  1. providing them with high-energy feed, and milking them regularly, Holstein cows are producing
  2. providing them with high-energy feed, and milked regularly, the Holstein cow produces
  3. provided with high-energy feed, and milking them regularly, Holstein cows are producing
  4. provided with high-energy feed, and milked regularly, the Holstein cow produces
  5. provided with high-energy feed, and milked regularly, Holstein cows will produce

Once again, another long, complicated, heavily punctuated sentence. Sensing a trend? Before we start breaking this sentence apart, however, let’s scan to see if there is anything a little easier to work with in this sentence by looking at the differences between the answer choices. We notice quite a few changes in tense (“providing” vs. “provided”, “milking” vs. “milked”, and “are producing” vs. “produces” “will produce”), but we should also notice something simple: the change from “cows” to “cow”. Whenever we see changes between singular and plural in answer choices, we should immediately look for issues with pronoun agreement. We find our pronoun in the non-underlined portion of our sentence: “For the farmer who takes care to keep them cool”.

Because this part of the sentence isn’t underlined, we know that the sentence must discuss “Holstein cows”, not “the Holstein cow”. We can eliminate B and D.

Now we can look at the structure of the sentence. We see that the sentence begins with a list of three things the farmer does to cause the Holstein cow to produce more milk. A list is just a series of three or more items, which means we need to start thinking about parallelism. Since the first list item is not underlined, we want to match the structure of the second and third items to the first.

Looking at A, we see that the farmer

  • takes care to keep them cool
  • providing them with high-energy feed
  • milking them regularly

So all of our list items involve verbs, but the -ing verbs in the second two list items don’t match any tenses in the first list item. Eliminate A.

Note: some might think that “providing … regularly” might act as a modifying phrase for “takes care to keep them cool”, making an -ing verb a good pick (as in “I quickly called the office, holding my breath and hoping they wouldn’t be closed”). However, it doesn’t make much sense that feeding and milking cows would describe keeping them cool. Similarly, the comma between “feed” and “milking” would not be grammatical outside of a list.

C sets up a similarly confusing list. The farmer keeps the cows

  • cool
  • provided with high-energy feed
  • milking them regularly

“Cool” and “provided” both act as adjectives to describe the cows, but now “milking” doesn’t fit the structure. We can eliminate C, leaving us with E, in which the farmer keeps the cows

  • cool
  • provided with high-energy feed
  • milked regularly

 

calf-433000_640

 

GMAT Sentence Correction Question 4

There are no legal limits, as there are for cod and haddock, on the size of monkfish that can be caught, a circumstance that contributes to their depletion through overfishing.

  1. There are no legal limits, as there are for cod and haddock, on the size of monkfish that can be caught, a circumstance that contributes to their depletion through overfishing.
  2. There are no legal limits on the size of monkfish that can be caught, unlike cod and haddock, a circumstance that contributes to depleting them because they are being overfished.
  3. There are legal limits on the size of cod and haddock that can be caught, but not for monkfish, which contributes to its depletion through overfishing.
  4. Unlike cod and haddock, there are no legal size limits on catching monkfish, which contributes to its depletion by being overfished.
  5. Unlike catching cod and haddock, there are no legal size limits on catching monkfish, contributing to their depletion because they are overfished.

Sentences that are fully underlined can be intimidating — we aren’t sure of anything in the sentence. However, this is often a blessing in disguise. Since there may be multiple errors in the sentence, there may be multiple opportunities for us to rule out answer options.

Scanning over our sentence, we notice that the sentence sets up a comparison between the existence of legal size limits for cod and haddock and the lack of legal size limits for monkfish. Comparison questions are a favorite on the GMAT since they test a couple things — parallelism and “like/unlike” vs. “as”. Like with a list or a series, things being compared must have parallel structure. This concept is pretty easy to test for on a question-by-question basis, but “like” vs. “as” is another one of the GMAT Sentence Correction rules on idiom and diction we need to have memorized: “like” is used to compare two objects (nouns), while “as” is used to compare two actions (verbs).

Looking at A we see our comparison is between

  • There are no legal limits … on the size of monkfish that can be caught

and

  • there are [legal limits] for cod and haddock

… which actually works fine — the two things being compared are parallel and are both verbs, which fits with the use of “as”. We can keep A and check our other answers.

B sets up a comparison using “unlike”, so we know we should be comparing two nouns. However, the first part of our sentence doesn’t change at all, giving us a comparison between

  • There are no legal limits … on the size of monkfish that can be caught

and

  • cod and haddock

So we have a parallelism issue, (comparing a lack of legal limits to a fish doesn’t make much sense), AND we have an issue with idiom and diction (“unlike” can’t compare a verb to a noun). We can eliminate B based on either.

Looking at D and E, our other “unlike” answer choices, we see similar issues. In D, we compare “there are no legal size limits on catching monkfish” with “cod and haddock“. In E, we compare “there are no legal size limits on catching monkfish” with “catching cod and haddock (where “catching” is a gerund, a type of noun). We can eliminate both.

This leaves us with A and C. C doesn’t use “like” or “as”, so we’re out of luck there. It does, however, use the word “which”, which (… get it) should ring some more idiom and diction bells. On the GMAT, the word “which” must refer to the closest noun. For example:

We ran to the store, which made us tired.

is something we might say colloquially — the act of running made us tired. However, on the GMAT, this sentence would indicate that the store itself made us tired.

Looking at our sentence, we see that “which contributes to its depletion by being overfished” refers to monkfish … so monkfish are contributing to their own depletion. That doesn’t make much sense. We can eliminate C, leaving us with A as the correct answer.

boy-909552_640

 

GMAT Sentence Correction Question 5

Today’s technology allows manufacturers to make small cars more fuel-efficient now than at any time in their production history.

  1. small cars more fuel-efficient now than at any time in their
  2. small cars that are more fuel-efficient than they were at any time in their
  3. small cars that are more fuel-efficient than those at any other time in
  4. more fuel-efficient small cars than those at any other time in their
  5. more fuel-efficient small cars now than at any time in

Unlike the monkfish question, this sentence is short, and the underlined portion is even shorter. This means we may not be working with much.

Reading the initial sentence, we should notice the phrase “more fuel-efficient … than”. Once again, we’re dealing with a comparison: the fuel efficiency of small cars in the past vs. the fuel efficiency of small cars in the present.

However, we might notice something sounds odd about the comparison in first sentence. Small cars are “more fuel efficient now than at any time in their production history“. Well … doesn’t “any time in their production history” include “now”? Meaning that small cars are more fuel efficient now than they are … also now?

The impossible comparison is yet another classic GMAT trap, though this one doesn’t show up quite as often as some of the others in these questions. When we compare one thing in a group to the rest of the things in the group, we need to make sure that we exclude the one thing from the group we compare it to, typically using the word “other”. For example:

Keisha scored higher on the the test than all of the students in her class.

is illogical. Keisha is one of the students in her class; this sentence tells us she scored higher than all of them — including herself. We can correct this by excluding Keisha from the class in our comparison:

Keisha scored higher on the the test than all of the other students in her class.

Looking back at our sentence, we can eliminate any answers that use the phrase “at any time”: A, B, and E.

Let’s look at the comparisons made in C and D.

C tells us that manufacturers make “small cars that are more fuel efficient than those [small cars] at any other time in production history”. So the small cars manufacturers make now are more efficient than the small cars they made before.

D tells us that manufacturers make “more fuel efficient small cars than those [manufacturers] at any other time in their production history”. This has a slightly different meaning. Now, the sentence tells us that the manufacturers make more fuel-efficient small cars, not that the small cars they make are more fuel-efficient. So with C, the manufacturers are increasing fuel efficiency, while in D, the manufacturers are increasing the number of cars.

When dealing with issues of meaning on Sentence Correction questions, we want the correct answer to capture the meaning intended by the original sentence. Ignoring the errors in the original sentence, we see that it conveys the idea of “more fuel-efficiency” not “more cars”, making D wrong and C correct.

 

auto-2546073_640

 

Key Takeaways: Learning From The Hardest Sentence Correction Questions

So what can the hardest GMAT Verbal questions teach us about GMAT Sentence Correction questions in general?

  1. It’s crucial to know the tricky GMAT Sentence Correction rules about idioms, diction, and sentence structure — the most difficult questions tend to target constructions that sound okay in casual conversation but don’t fly on the GMAT.
  2. The hardest Sentence Correction questions often have complex sentence structures. Simplifying sentences by eliminating unnecessary elements (descriptive clauses and phrases, for example) can make them easier to decipher.
  3. Often dealing with several types of errors within a sentence gives us multiple opportunities to eliminate the wrong answer options. This makes long underlined portions a blessing in disguise.
  4. That said, challenging questions often come down to issues of meaning. Knowing GMAT Sentence Correction rules is usually enough to get you down to two answer choices, but on the hardest GMAT questions, you’ll usually need to understand what the sentence is saying to definitively answer the question.

 

What’s Next?

What are the most important grammar rules for the GMAT? The most common GMAT idioms? The top tips for Sentence Correction? These articles expand on the concepts used in these five problems, explaining what you need to memorize for Sentence Correction before test day.

Aiming for an 800 on the GMAT? This article shares key strategies for getting a perfect score.

If you’d like similar analyses of the hardest questions from other GMAT question types, check out our five hardest Data Sufficiency questions (coming soon).

The post The 5 Hardest GMAT Sentence Correction Questions appeared first on Online GMAT Prep Blog by PrepScholar.

]]>
8 Expert Tips for GMAT Critical Reasoning Questions /gmat/blog/gmat-critical-reasoning-tips/ Sat, 06 May 2017 23:00:33 +0000 http://gmat.psblogs.com/?p=1226 Critical reasoning questions often worry GMAT test-takers. What exactly are GMAT critical reasoning questions, and what skills do you need to develop to answer them correctly? How can you best prepare for critical reasoning questions before the exam? In this article, I’ll go over the five main critical reasoning question types and examples of each … Continue reading "8 Expert Tips for GMAT Critical Reasoning Questions"

The post 8 Expert Tips for GMAT Critical Reasoning Questions appeared first on Online GMAT Prep Blog by PrepScholar.

]]>
reasoning1

Critical reasoning questions often worry GMAT test-takers. What exactly are GMAT critical reasoning questions, and what skills do you need to develop to answer them correctly? How can you best prepare for critical reasoning questions before the exam?

In this article, I’ll go over the five main critical reasoning question types and examples of each one, as well as plenty of GMAT critical reasoning tips on how to ace these question types come exam day.

 

GMAT Critical Reasoning Overview

Critical reasoning questions appear in the verbal section of the GMAT, alongside sentence correction questions and reading comprehension questions. There are 36 questions in total on the verbal section, and around 11 of them are critical reasoning questions.

GMAT critical reasoning questions test your ability to evaluate and draw conclusions based on logical arguments. You won’t need any formal training in logic or rhetoric to do well on critical reasoning questions. However, you will need to be able to analyze an argument and unpack it into its various parts.

 

The 5 GMAT Critical Reasoning Question Types

There are five main critical reasoning question types: Strengthen the Argument, Weaken the Argument/Find the Flaw, Inference, Draw a Conclusion, Find the Assumption, and Paradox/Discrepancy. Let’s go through each question type. For each question type, we’ll go over what the question will be asking you to do, key words associated with that type, how each it will be worded, and an example question.

 

Question Type 1: Strengthen the Argument

Strengthen the Argument questions will ask you to choose the statement or piece of evidence that would support, bolster, or add to the argument in the given passage. Start by figuring out the main argument of the passage. Then choose the answer choice that provides the best evidence for that argument.

Key words you might see are support or strengthen. Clue words can help you figure out what category a question falls into, which will help you know how to approach the question.

A Strengthen the Argument question may be worded as:

Which of the following, if true, most strongly supports…?

Which of the following, if true, would most strengthen…?

The statements above, if true, best support which of the following assertions?

Example:

supports

The answer you choose for a Strengthen the Argument question should be directly relevant to the specific argument in question and should provide a specific piece of information that justifies the argument (in this case, the Write Company’s assertion that their plan will lead to an increase in its sales of pencil leads).

Answer choice A doesn’t provide enough directly relevant info; we are told that the Write Company’s pencil leads are more expensive than other manufacturers’, but don’t know whether their pencils themselves are more or less expensive than average.

Answer choice B is too general, as it addresses annual sales of all mechanical pencils and not Write Company sales specifically.

The passage tells us that the new lead will be sold at the same price as the Write Company’s current lead, so answer choice C doesn’t tell us anything about the potentiality of increased sales.

Answer choice D tells us only about a rival manufacturer, which may or may not have an impact on the Write Company’s sales.

Answer choice E is correct, as this test marketing information strongly supports the Write Company’s projection of the success of its new plan.

 

Unfortunately, this won't be one of your answer choices.
Unfortunately, this won’t be one of your answer choices.

 

Weaken the Argument/Find the Flaw

Weaken the Argument questions will ask you either to 1) select the fact or piece of evidence that would most effectively undermine the given argument or 2) to find the logical flaw in the given argument’s reasoning. If you know you’re being asked a Weaken the Argument question, start by figuring out (and perhaps jotting down) what exactly that argument is. Then look at each answer choice carefully. If true, does it create a crack in the argument’s foundation?

Key words you might see are weaken or doubts.

A Weaken the Argument question may be worded as:

Which of the following, if true, most seriously weakens the argument?

Which of the following, if true, would cast the most doubt on…?

The objection implied above…is based on doubts about…?

Which of the following points to the most serious logical flaw in the author’s argument?

Example:
weaken
Before answering this question or looking at the answer choices, consider: What would weaken the argument? The conclusion that the statue is a forgery is based on the premise that “an ancient surface usually has uneven weathering” and this one does not. Thus, if the uniform surface was caused by some legitimate force (whether natural or man-made), the argument will be weakened.

Answer choice A doesn’t weaken the argument because the argument concerns whether or not the statue is a forgery, not how the museum obtained it.

Answer choice B fails to take into account the fact that any forger would likely imitate the most common features of Greek statues of the time period.

Answer choice C is correct. It gives a possible legitimate explanation for the statue’s uniform surface.

Answer choice D isn’t relevant because the statue under discussion has a uniform surface, not a weathered one.

Answer choice E provides an example that strengthens the argument rather than weakening it.

 

Can you spot the crack in the argument?
Can you spot the crack in the argument?

 

Inference

Inference questions ask you to make inferences—draw logical conclusions—based on the evidence in the given passage.

Key words you might see are imply or infer.

An Inference question may be worded as:

Which of the following can properly be inferred from the statements above?

Which of the following can be correctly inferred from the statements above?

Example:

inference For inference questions, you must draw conclusions only from the information you’re directly given in the passage. Beware of answer choices that contain words like ‘any,’ ‘best,’ ‘worst,’ ‘only,’ ‘all,’ or ‘none,’ as they are often overly general and can’t be verified by the limited information in the passage. Incorrect answer choices will often be overly extreme, make leaps in logic that can’t be verified based on the given info, or contain tangential/unrelated information.

Answer choice A only references underweight individuals, while the passage only specifically mentions overweight individuals, so we can’t make this inference based on the given information.

Answer choice B is incorrect. The passage only mentions the possibility of increasing HDL levels for some individuals through exercise and weight reduction. It doesn’t mention the individuals’ age or who is high-risk for high cholesterol levels.

Answer choice C makes a too-far leap in logic, generalizing about all human beings, while the passage only mentions the benefits of exercise and weight reduction for some. It also can’t be known from the passage whether exercise and weight reduction are the only or best methods of lowering cholesterol levels.

Answer choice D is correct. Since increased HDL levels decrease bloodstream cholesterol levels, and exercise and weight reduction increase HDL levels for some individuals, we can infer that regular exercise and weight reduction lowers cholesterol levels in the bloodstream of some individuals.

Answer choice E generalizes that ‘only’ exercise is necessary to decrease cholesterol levels. The passage doesn’t say that exercise is the only means of decreasing cholesterol levels, and it doesn’t mention the relative success of exercise programs for individuals of certain weights.

 

Inference questions ask you to look at the information presented in the given argument closely.
Inference questions ask you to look at the information presented in the given argument closely.

 

Find the Assumption

Find the Assumption questions will ask you to select the answer choice with the information that must be true (the ‘assumption’) in order for the given argument to be accurate. To figure out the answer, you’ll first need to figure out the main thrust of the given argument. The correct answer choice, again, will have to be true in order for that argument to be logical. Incorrect answer choices will often be possibly true, but won’t be absolutely necessary to the argument’s validity.

Key words you might see are assumption, because, or rely.

A Find the Assumption question may be worded as:

Which of the following is an assumption made in drawing the conclusion above?

Which of the following in an assumption on which the argument relies?

Example:

assumption

Answer choice A is too absolute. We can’t know for sure that a hiring program will always be successful if it includes interviews.

Answer choice B is off topic. The developments of job descriptions are not relevant to the argument.

Answer choice C is correct. For the interview to be an essential part of a successful hiring program, the interviewer must be able to accurately identify applicants whose personalities are unsuited to the job in question.

Answer choice D goes too far in its assumption. Note that extreme word ‘only.’

Answer choice E is incorrect—we can’t know this from the argument. We don’t know anything about past priorities in hiring decisions.

 

Learning to unpack arguments and identify premises, assumptions, and conclusions is an important skill for answering critical reasoning questions.
Learning to unpack arguments and identify premises, assumptions, and conclusions is an important skill for answering critical reasoning questions.

 

Paradox/Discrepancy

Paradox/Discrepancy questions will ask you to choose the answer choice that explains the paradox in the given argument. A paradox, in the context of the GMAT, refers to the coexistence of two seemingly contradictory pieces of information. The correct answer choice will logically explain why those pieces of information are not actually contradictory.

Key words you might see are explain, paradox, or discrepancy.

A Paradox question may be worded as:

Which of the statements below provides the most likely explanation for the two seemingly contradictory statements above?

Which of the following, if true, most helps to resolve the paradox outlined above?

Which of the following, if true, best explains the reason for the apparent discrepancy described above?

Example:

paradox The paradox here is that while brand name products are not actually higher quality than nonbrand rival ones, they are at a bigger marketing advantage than ever. What information would resolve this paradox?

Answer choice A is correct and resolves the paradox. If brand names are believed by customers to be a guarantee of high quality, that explains their marketing advantage over rival brands.

Answer choice B wouldn’t resolve the paradox: If consumers understand that a brand name isn’t a guarantee of quality over time, the marketing advantage of a brand name should decrease rather than continue to increase.

Answer choice C is incorrect because corporations’ interest in brand names does not explain their marketing advantage.

Answer choice D doesn’t explain the paradox because the difficulty or ease of establishing a brand name isn’t relevant to its relative marketing advantage.

Answer choice E is also irrelevant: Transferring to a new advertising agency is not sufficient as an explanation for the marketing advantage experienced by all brand names.

 

Paradox questions ask you to resolve apparent contradictions, discrepancies or disparities.
Paradox questions ask you to resolve apparent contradictions, discrepancies or disparities.

 

Top GMAT Critical Reasoning Tips

Let’s go through five of the top GMAT critical reasoning tips to help you do your best on this tricky question type.

Pace yourself properly. You will have 65 minutes to complete the verbal section. GMAT critical reasoning questions, in general, don’t take quite as much time as the average reading comprehension question, but more time than a sentence correction question. As you work through your critical reasoning GMAT practice, you should plan to spend about a minute and a half on each one.

Read the question first. It can be helpful to read the question before you read the rest of the passage. Try to figure out which question type it is. This will help you figure out exactly what you should be looking for in the answer choices. For paradox questions, for example, you’re looking for a new piece of information that resolves a ‘problem’ or contradiction, whereas in an inference question you’re looking for a conclusion that can be reasonably drawn from the given information.

Think carefully about what you’re being asked. As soon as you read the question, try to discern what’s exactly is being asked of you. Are you being asked to draw a conclusion based on the given information? To reconcile two contradictory statements? This will determine how you approach the answer choices. For example, if you’re being asked to choose a statement that would weaken the argument presented in the passage, you’ll need to start off by 1) identifying the main argument of the passage and 2) select the answer choice that contradicts or provides an exception to that argument. If you’re being asked to select an answer choice that supports the main argument, you’ll still need to complete the first step (identifying the passage’s argument), but step 2 will be to select an answer choice that bolsters that argument.

Avoid extreme answer choices. Extreme or absolute answer choices are a red flag. Try to start taking notice of “clue words” in your critical reasoning GMAT practice. For the most part, avoid answer choices that contain language like ‘only,’ ‘never,’ ‘always,’ ‘all,’ ‘none,’ ‘best,’ ‘worst,’ ‘must,’ etc. They’re nearly always incorrect. For example, in answer choice C in the inference example above, “Exercise and weight reduction are the most effective methods” is a tip-off that the answer choice is likely wrong. Think about it: The passages you’re given in these questions are brief, and the accompanying questions ask you to do some nuanced critical thinking, so sweeping, broad generalizations are pretty unlikely candidates for correct answer choices.

Steer clear of answer choices with information that wasn’t directly given in the passage. This is especially true for inference questions. Everything you need to answer the question will be right there for you in the passage. If an answer choice is off topic or addresses a tangential issue (for instance, if an argument addresses the link between weight and age and an answer choice references the effects of exercise on weight), it might be a red flag.

Take note as you work through GMAT critical reasoning practice questions: Sometimes you’ll get new pertinent information in an answer choice (for example, the correct answer to a paradox question will provide an explanation for an apparent contradiction), but incorrect answer choices will contain information that’s not directly related to the passage at all or provide unrelated new information that doesn’t answer the question.

 

Pace yourself well on critical reasoning questions! Not answering every question will affect your score.
Pace yourself well on critical reasoning questions! Not answering every question will affect your score.

 

How to Prepare for GMAT Critical Reasoning Questions

Practice identifying premises, assumptions, and conclusions. Every argument has a premise (the foundational underlying logic of an argument, including the evidence behind a conclusion), one or more assumptions (information that must be true in order for the ultimate conclusion to be true) on which it’s based, and one or more possible conclusions (the statement that the premise supports). As you complete GMAT critical reasoning practice questions, label the premises, assumptions, and conclusions of each argument. This will help you learn to isolate each one quickly, which will improve both your timing and performance in critical reasoning questions.

Learn to identify the question types as you complete GMAT critical reasoning practice questions. Identifying the critical reasoning question types as you practice will help you decide how to approach them. To do this, learn the ‘clue words’ and most common question stems for each question type.

Practice with realistic GMAT critical reasoning questions and hone in on your weaknesses. Use the free GMATPrep software or the resources in our GMAT Verbal Practice article to complete a variety of GMAT critical reasoning practice questions. Keep track of how many you get wrong of each question type, and focus on those in your prep time.

 

Hone in on your weaknesses to maximize the efficacy of your prep!
Hone in on your weaknesses to maximize the efficacy of your prep!

 

What’s Next?

If you’d like similar breakdowns of other GMAT question types, check out our top tips for data sufficiency and sentence correction questions.

Want to find out more about the GMAT format overall, as well as get more GMAT critical reasoning tips? Our complete guide to the GMAT format will give you a comprehensive overview of each section.

Our collection of GMAT sample questions is a helpful resource if you’re looking for more ways to practice before the exam (including more critical reasoning GMAT practice questions).

The post 8 Expert Tips for GMAT Critical Reasoning Questions appeared first on Online GMAT Prep Blog by PrepScholar.

]]>
8 Top Tips for GMAT Sentence Correction Questions /gmat/blog/gmat-sentence-correction-tips/ Fri, 05 May 2017 21:00:23 +0000 http://gmat.psblogs.com/?p=1241 What are GMAT sentence correction questions? What skills and concepts do they test? What are the most effective GMAT sentence correction tips? In this article, I’ll go over the format of sentence correction questions on the exam, the grammar concepts you’ll need to know to complete them, a few examples of the most common sentence correction error … Continue reading "8 Top Tips for GMAT Sentence Correction Questions"

The post 8 Top Tips for GMAT Sentence Correction Questions appeared first on Online GMAT Prep Blog by PrepScholar.

]]>
correcting-1870721_1280

What are GMAT sentence correction questions? What skills and concepts do they test? What are the most effective GMAT sentence correction tips?

In this article, I’ll go over the format of sentence correction questions on the exam, the grammar concepts you’ll need to know to complete them, a few examples of the most common sentence correction error types, and how best to prepare for these tricky questions before you take the GMAT.

 

The Basics: GMAT Sentence Correction Questions

Sentence correction questions are part of the verbal section of the GMAT. The verbal section contains 36 questions (including sentence correction, reading comprehension, and critical reasoning questions), and usually around 13 are sentence correction questions.

A sentence correction GMAT question will present you with a sentence, part or all of which will be underlined. Beneath the sentence will be five answer choices, each of which presents a different way to replace the underlined part of the sentence. The first of the answer choices will always be identical to the underlined portion. The next four will be different.

These questions test your ability to use English accurately and effectively, so the answer you select should be both grammatically correct and the most effective (concise, clear, and not awkwardly phrased) of the choices. The GMAT tests you in very specific ways, so it’s important to be familiar not only with grammar rules, but with the GMAT sentence correction rules in particular.

 

Identifying the error type is the first step to correcting a sentence on the GMAT.
Identifying the error type is the first step to correcting a sentence on the GMAT.

 

What Do GMAT Sentence Correction Questions Test?

If you’re worried about learning GMAT sentence correction rules, fear not: they only test a few basic grammar concepts and skills. On a given test, you’ll generally see multiple questions about each of the topics.

There are seven main error types you’ll find on GMAT sentence correction questions.

 

Subject-Verb Agreement 

GMAT sentence correction questions will require you to locate the subject of a sentence and its verb and to ensure that they match: a plural subject goes with the plural form of a verb, and a singular subject goes with the singular form of a verb.

In many sentences, the subject and its verb are right next to each other. In the sentence “He ran for mayor,” for example, the subject is ‘he’ and the verb, ‘ran.’ In sentence correction questions on the GMAT, locating the subject and its verb will be a bit trickier, usually because the subject and verb won’t be next to each other in a sentence. Take a look at this example, and try to find the main subject and verb:

Attempts to assuage his fear and soothe his conscience fails miserably, as we soon discover.

The subject of the verb ‘fails’ is ‘attempts.’ This is incorrect: ‘Attempts’ is a plural subject, while ‘fails’ is the singular form of the verb ‘to fail.’ (Remember that the singular form of most verbs ends in -s.) The corrected version would be:

Attempts to assuage his fear and soothe his conscience fail miserably, as we soon discover.

A GMAT sentence correction question will use the words in between the subject and the verb (in the case of this example, ‘to assuage his fear and soothe his conscience’) to confuse you or throw you off track. A good clue that a sentence correction question is asking you to correct a subject-verb agreement error is that the answer choices include both singular and plural forms of the same verb.

 

Reviewing parts of speech can help you prepare for subject-verb agreement questions.
Reviewing parts of speech can help you prepare for subject-verb agreement questions.

 

Verb Tense Accuracy/Consistency

In general, sentence correction questions will ask you to keep verb tense consistent through a sentence (past tense generally goes with past tense, for example). Sometimes, though, you might need to shift the verb tense if it’s specifically required by a sentence’s meaning, as in these examples:

We were cleaning up hurriedly when our parents suddenly arrive.

What’s wrong here? There was an ongoing activity happening in the past (‘were cleaning up’) that was interrupted, also in the past, by the parents’ arrival. In this case, the verb tense needs to shift because of a sudden change in activity. The correct version would be:

We were cleaning up hurriedly when our parents suddenly arrived.

To know how and when to shift the verb tense, look for clues related to time like ‘suddenly,’ ‘until,’ ‘already,’ and ‘by the time.’

An example of a correct verb tense shift:

Amal was always a good student, and he is still quite studious today.

In this case, the verb tense shifts from past to present to indicate what Amal is/was like at different points in time.

 

Being aware of how time functions in a sentence will help you with verb tense questions.
Being aware of how time functions in a sentence will help you with verb tense questions.

 

Idioms

Idioms are simply common ways of saying things in English, often having to do with prepositions. We say someone graduated ‘from’ college, for example, not ‘to’ college. John is worried about his finances because he doesn’t have ‘much’ money, not because he doesn’t have ‘many’ money. I have ‘less’ cake after the party, not ‘fewer’ cake.

Idioms come up frequently in GMAT sentence correction questions and can be especially difficult for non-native English speakers. It can be helpful to memorize a few of the most common ones that tend to crop up on the GMAT, but some students memorize lists of 200+ idioms. While this may make you feel better, it probably won’t help you improve your score too much: There will only be a few questions on the GMAT that require you to be familiar with idioms, and they’ll be different on every exam, so your prep time is likely better spent elsewhere. Just note the idiomatic expressions with which you’re not familiar as you practice, keep a running list, and become familiar with those.

 

shield-1519642_1280
Idioms are common ways of phrasing things in English.

 

Illogical Comparisons

Some GMAT sentence correction questions will ask you to correct an illogical comparison, or a sentence that compares two unlike things (for example, a noun to a verb, or two inappropriately matched nouns). For example, if you say:

He loves donuts more than me.

It’s actually quite a sad sentiment, as you’re saying that someone is fonder of donuts than they are of you. Obviously, what you really want to do is compare your love of donuts to his love of donuts. Correct versions of the sentences would read, “He loves donuts more than I love donuts,” or “He loves donuts more than I do.”

Illogical comparison questions are quite easy to spot, as they will usually include clue comparison phrasing like ‘more than,’ ‘less than,’ ‘like’ or ‘unlike,’ or ‘as/as.’ Start to take note of these clue words as you complete GMAT sentence correction practice questions.

 

Donuts ARE pretty rad, though.
Donuts ARE pretty rad, though.

 

Parallelism

Parallelism means what it sounds like: you want the related parts of your sentence to be parallel, or matching. This error is especially common in lists. Here’s a simple example of a parallelism error:

I loved to sing, to dance, and acting.

The first two items in the list are infinitive verbs (verbs preceded by ‘to’)—to sing and to dance—followed by a noun (‘acting’). The correct version of the sentence would read, “I loved singing, dancing, and acting,” or “I loved to sing, to dance, and to act.”

 

Parallelism questions ask you to make sure the appropriate parts of a sentence are parallel, or 'match.'
Parallelism questions ask you to make sure the appropriate parts of a sentence are parallel, or ‘match.’

 

Dangling/Misplaced Modifiers

Modifiers are words or phrases (in the case of the GMAT, we’re referring specifically to phrases) that modify, or describe/alter/specify, another part of a sentence. Here’s a correctly placed modifier:

Looking around fearfully, she darted into the building before anyone could see her.

Here, the modifier is ‘looking around fearfully,’ which describes ‘she.’

On the GMAT, you will encounter modifiers that aren’t in the right place. These are called dangling or misplaced modifiers. In general, remember that you want your modifier to be as close to the noun it’s modifying as possible.

A dangling modifier is one that ‘hangs’ off a sentence (usually the beginning) and doesn’t modify the right word or words. For example:

A majestic fairy tale creature, Frankenstein admired the Pegasus for her intellect.

There are many ways to describe Frankenstein, but ‘majestic fairy tale creature’ is probably not one of them. That modifying phrase should be next to the word it’s modifying (Pegasus). The new sentence would read:

Frankenstein admired the Pegasus, a majestic fairy tale creature, for her intellect.

A misplaced modifier can be a little more difficult to spot. Here’s an example:

She made her life plan to write a bestselling novel on the day she graduated from college.

This sentence might be true, but it’s rather illogical and more than a bit unrealistic. Here, the modifying phrase is ‘on the day she graduated from college,’ which is when she likely did the planning, not the day she did the writing. Right now, it’s misleading. We’d rewrite it as such:

On the day she graduated from college, she made her life plan to write a bestselling novel.

 

I've heard from Frankenstein that she's very smart.
I’ve heard from Frankenstein that she’s very smart.

 

Pronoun Usage

Pronouns stand in for nouns, and the nouns they stand in for are called antecedents. For example, in the sentence “Carol left her coat outside,” Carol is the antecedent of the pronoun her.

The most common pronoun error type on the GMAT is the ambiguous pronoun. An ambiguous pronoun is one whose antecedent is unclear:

Sarah and Julie took her dog for a walk.

In this sentence, whose dog is it? What is the antecedent of ‘her?’ Sarah or Julie? There’s no way to tell for sure what the antecedent of ‘her’ is. The correct version of the sentence would clarify this for the reader (for example, “Sarah and Julie took Julie’s dog for a walk”).

On the GMAT, look out particularly for pronouns like ‘it’ and ‘they.’ While any pronoun can be ambiguous, these are the two most likely culprits. For example:

When George dropped his vase onto the glass table, it cracked.

What cracked? The vase or the table? We can’t be sure, because we don’t know what the antecedent of ‘it’ is. We need to clarify that for the reader.

Pronoun agreement errors are also common on the GMAT. A pronoun agreement error is when a pronoun’s antecedent is plural, while the pronoun is singular, or vice versa. For example:

The male peacock spreads their feathers to impress the female peacock.

The singular antecedent ‘the male peacock’ does not agree with the pronoun ‘their.’ Instead, the sentence should be written as such:

The male peacock spreads its feathers to impress the female peacock.

This error type comes up particularly often in sentences dealing with animals. When speaking of an animal in general (‘the brown bear,’ ‘the flamingo,’), the correct pronoun is ‘it.’

 

Don't leave your pronouns a mystery. Clarify them!
Don’t leave your pronouns a mystery. Clarify them!

 

Clarity and Concision

GMAT sentence correction questions will require you not only to make sure that a sentence is grammatically correct, but also to ensure that it’s not redundant, ambiguous, awkwardly phrased, or clunky.

For example, take a look at this sentence:

At this point in time, she has knowledge and comprehension of her error.

What could be more concise in this sentence? Let’s rewrite it.

Now, she understands her error.

Notice that the two sentences mean the same thing, but the former uses unnecessary phrases like “at this point in time” when simply “now” is sufficient. When you think you have found a clarity and concision error, take note: Does the shorter sentence shift the sentence’s meaning? Or does it simply say the same thing in a cleaner and more concise way?

Remember that this doesn’t mean you should simply choose the shortest answer choice or that the shortest answer option is always the correct one. You should always check for other error types first, and eliminate answer choices that way. If you can whittle the answer options down to two, however, it’s a safe bet to choose the shorter one.

 

When it comes to the GMAT, shorter sentences are often preferable.
When it comes to the GMAT, shorter sentences are often preferable.

 

GMAT Sentence Correction Practice Questions: Examples

Let’s go through four examples of GMAT sentence correction question, covering the most common error types.

 

Practice Question 1: Subject-Verb Agreement

While larger banks can afford to maintain their own data-processing operations, many smaller regional and community banks are finding that the cost associated with upgrading data-processing equipment and with the development and maintenance of new products and technical staff are prohibitive.

(A) cost associated with

(B) costs associated with

(C) costs arising from

(D) cost of

(E) costs of

Source: http://www.mba.com/us/the-gmat-exam/gmat-exam-format-timing/verbal/sample-sentence-correction-question.aspx

This is a subject-verb agreement question; one clue is that the various answer choices include both plural and singular subject options (‘costs’ and ‘cost’). The first thing to do is to locate the subject and verb: ‘costs’ or ‘cost’ is the subject, while ‘are’ is the verb. Since we don’t have the option to change the plural verb ‘are,’ the subject must be ‘costs,’ so we can narrow down the answer choices right away to B, C, or E. Choices A and D contain subject-verb agreement errors.

Answer choices C and E contain idiomatically incorrect expressions (‘arising from’ and ‘costs of upgrading’).

Answer choice B (‘costs associated with’) is correct in terms of both subject-verb agreement and idiomatic expression.

 

Matching a subject to its corresponding verb is the first step to any subject-verb agreement question.
Matching a subject to its corresponding verb is the first step to any subject-verb agreement question.

 

Practice Question 2: Dangling Modifier

dangling modifier 2

The original sentence is an example of a dangling modifier: In answer choice A, ‘architects and stonemasons’ is incorrectly modifying ‘huge palace and temple clusters.’ So some of the other answer choices are likely to contain dangling modifiers, too. When you see a phrase followed by a comma introducing a sentence, it’s a good idea to check for a dangling modifier before doing anything else.

Answer choice B is also a dangling modifier (‘architects and stonemasons’ is incorrectly modifying ‘without of benefits of animal transport’).

Answer choice D is also a dangling modifier and contains clarity and concision errors, beginning with the passive construction ‘there were built.’

Answer choice E isn’t a dangling modifier, but it’s still incorrect: It contains the grammatically incorrect ‘were the Maya who.’

Answer choice C corrects the modification error: ‘Architects and stonemasons’ is correctly modifying ‘the Maya.’ It is also preferable because it contains the active verb ‘built’ rather than the passive ‘were built,’ which is in several of the other answer choices. Answer choice C (‘the Maya…’) is correct.

 

pencil-918449_1280
In dangling modifier questions, you’re looking to match the correct noun with its appropriate modifying phrase.

 

Practice Question 3: Illogical Comparison, Idioms, Clarity and Concision

Many sentence completion questions contain more than one error. Several error types apply to this question.

dangling modifier

The original sentence, answer choice A, is awkwardly phrased and wordy (a clarity and concision issue) and contains an illogical comparison (‘computer skills’ to ‘disinclination’). Look for illogical comparison issues when you see clue words like ‘as,’ ‘more,’ ‘less,’ ‘than,’ or ‘like.’

Answer choice B illogically compares ‘computer skills or other technical skills’ to ‘many people.’

Answer choice C has clarity and concision errors; the phrase ‘analytical skills bring out a disinclination’ is idiomatically incorrect. Answer choice E contains the wordy construction ‘Many people have a disinclination.’

Answer choice D clearly compares ‘computer skills or other technical skills’ to ‘analytical skills.’ It is also clearer and more concise than the other choices. The correct answer is D (“Many people, willing…”).

 

When it comes to the GMAT, don't compare apples and oranges!
When it comes to the GMAT, don’t compare apples and oranges!

 

Practice Question 4: Pronoun Usage, Clarity and Concision

This question contains both pronoun usage and clarity/concision problems.

sentence correction

Answer choices A and B are both unclear and too wordy, but answer choice B also contains a pronoun agreement error. ‘Crab,’ a singular noun, is not the correct antecedent for the plural ‘their.’ It’s a good idea to look for pronoun usage problems when you see potentially problematic pronouns like ‘it’ or ‘their’ in the original sentence or an answer choice, or when you see several different options for pronouns in the answer choices.

Answer choice E also contains a pronoun agreement error: ‘Crab’ is a singular antecedent, while ‘they’ is a plural pronoun.

Answer choice D contains an idiomatic error (‘because of living’ is not a correct idiomatic expression).

Answer choice C corrects the pronoun error and appropriately matches ‘crab’ with the singular pronoun ‘it.’ The correct answer is C (‘because it lives’).

 

Clarity is the name of the game for sentence correction questions.
Clarity is the name of the game for sentence correction questions.

 

Sentence Correction GMAT Question Tips: Key Strategies

Remember that the first choice is “no change.” This is a simple tip, but it will save you a bit of reading time. It’s not labeled as no change, but the first answer choice is always the same as the given sentence. Try to identify the potential error type as soon as you read it (an introductory phrase followed by a comma, for example, often signals a dangling modifier). If you can’t, quickly skim the other answer choices for clues.

Shorter is often better. Particularly if you’re down to two answer choices in your process of elimination, choose the shorter option! Remember, the GMAT loves concision. If you’re fairly certain that two options are technically (grammatically) correct, the shorter answer choice is a safe bet. The shortest answer won’t always be the right one, but it’s a good guideline for clarity and concision questions in particular. Does one answer choice say the same thing as another, but uses extraneous words? There’s a good chance it’s not the best choice.

Cross out extraneous words and phrases. This is something you can use your scratch pad (or, at home, your sentence correction worksheet) for. Particularly for subject-verb agreement questions, try rewriting the sentence and crossing out all the extra words between the subject and verb. Prepositional phrases between the subject and verb, for example, are often used to confuse you. Isolating the key words will cut down on confusion and potential timing issues. It will also help you eliminate incorrect answer choices more confidently.

Take about a minute per question. Since you have 65 minutes to complete the verbal section and 36 questions to complete, and reading comprehension and critical reasoning questions tend to take a bit longer (one and a half to two minutes each), you should plan to spend around one minute on each sentence correction question. You should keep a sentence correction worksheet to log the amount of time you spend on each question.

 

Practice crossing out extraneous words in sentences on the GMAT.
Practice crossing out extraneous words in sentences on the GMAT.

 

GMAT Sentence Correction Tips: How to Prepare

Brush up on basic grammar terms, rules, and concepts. You won’t be tested directly on grammar terms (such as parts of speech) on the GMAT, but it can be very helpful to review them before the exam (especially if you’re rusty) as part of your GMAT sentence correction practice. If you’re thinking in terms of subjects and verbs when you’re faced with a subject-verb agreement question or can quickly tell the difference between the past perfect and the present perfect for a verb tense question, it becomes a lot easier to quickly correct any technical errors in a given sentence. Remember that you need to focus specifically on GMAT sentence correction rules, not just on grammar rules in general.

Learn to recognize error types. Recognizing sentence correction error types quickly and accurately will improve both your timing and your performance. GMAT sentence correction practice should always include labeling error types as you go along.

Once you recognize an error type, you’ll know much more quickly what to look for and how to solve the problem. For example, a subject-verb agreement question means you’re looking to isolate the subject and verb and eliminate the answer choices that match plural subjects with singular verbs, or vice versa, right off the bat. A dangling modifier question will require you to look for the noun immediately following a modifying phrase to make sure it’s being modified accurately. Knowing how to tackle each kind of error type will help you streamline your approach to each question. Use the list above and label each sentence correction question as you review. With practice, you’ll learn to recognize them right away and apply GMAT sentence correction rules to your approach to each question.

Practice with real sentence correction GMAT questions regularly. During GMAT sentence correction practice sessions, use the GMATPrep software or the resources in GMAT Verbal Practice to practice specifically with sentence correction questions and review them. This will help you get comfortable with the format of each error type and with the various GMAT sentence correction rules. As soon as possible, as you complete GMAT sentence correction practice questions, hone in on your weakest areas and focus your prep time on them. Keep track of your most common errors on a sentence correction worksheet.

Learn the most common idioms. Especially if you’re not a native English speaker, consider learning the most common idioms that crop up on the GMAT and incorporating that work into your GMAT sentence correction practice. Keep a running list of idioms that aren’t familiar to you as you practice, and make sure you know those as well.

 

whats-next-1462747_1280

What’s Next?

If you’d like similar breakdowns of other GMAT question types, check out our top tips for data sufficiency and critical reasoning questions.

For a comprehensive overview of the GMAT and how to prepare for each section, including more GMAT sentence correction tips and GMAT sentence correction practice questions, go to our complete GMAT study guide.

Nervous about the rest of the GMAT verbal section? Our complete guide to GMAT verbal will help you ace the reading comprehension and critical reasoning portions of the exam, in addition to offering even more GMAT sentence correction tips.

The post 8 Top Tips for GMAT Sentence Correction Questions appeared first on Online GMAT Prep Blog by PrepScholar.

]]>